Download as pptx, pdf, or txt
Download as pptx, pdf, or txt
You are on page 1of 363

PULMONARY

MEDICINE
Joselito R. Chavez, MD
Aaron Joseph K. Tiu, MD
Topics to be covered (PCP Glossary of
Diseases)
• Internist capable of total • Internist expected to diagnose,
management may refer for definitive treatment
• Pneumonia (CAP, HCAP) • Occupational lung disease
• Internist expected to diagnose • Obstructive sleep apnea
and initiate treatment • Additional topics
• Bronchial asthma • Bronchiectasis
• COPD • Lung abscess
• Pleural effusion • GINA CPG
• Venous thromboembolism • GOLD CPG
• ARDS • VTE CPG (CHEST)
Pneumonia
1. 54/M was suspected to have HCAP. He was
previously admitted in PGH for initiation dialysis and
has been on chronic dialysis since then. What is the
likely organism? (HPIM 19th Ch153, p.803)
A. Pseudomonas aeruginosa
B. Acinetobacter spp.
C. Methicillin resistant Staphylococcus aureus
D. MDR Enterobacteriaceae
1. 54/M was suspected to have HCAP. He was
previously admitted in PGH for initiation dialysis and
has been on chronic dialysis since then. What is the
likely organism? (HPIM 19th Ch153, p.803)
A. Pseudomonas aeruginosa
B. Acinetobacter spp.
C. Methicillin resistant Staphylococcus aureus
D. MDR Enterobacteriaceae
2. The most common mechanism by which
microorganisms gain access to the lower
respiratory tract (HPIM 19 Ch153, p.804):
th

A. Inhalation of contaminated droplets


B. Aspiration from the oropharynx
C. Hematogenous spread
D. Contiguous extension
2. The most common mechanism by which
microorganisms gain access to the lower
respiratory tract (HPIM 19 Ch153, p.804):
th

A. Inhalation of contaminated droplets


B. Aspiration from the oropharynx
C. Hematogenous spread
D. Contiguous extension
Pneumonia – Pathophysiology
• Access to the lower respiratory tract:
• aspiration from the oropharynx – most common
• inhalation of contaminated droplets
• hematogenous spread (e.g., from tricuspid endocarditis)
• contiguous extension from an infected pleural or mediastinal space
• Clinical pneumonia manifests only when the capacity of the alveolar
macrophages to ingest or kill the microorganisms is exceeded
Pneumonia – Pathophysiology
• Host inflammatory response rather than proliferation of
microorganisms triggers the clinical syndrome of pneumonia
• Release of inflammatory mediators such as IL-1 and TNF results in
fever
• Chemokines such as IL-8 and granulocyte colony-stimulating factor
stimulate release of neutrophils and their attraction to the lung 
peripheral leukocytosis and increased purulent secretions
Pneumonia – Pathophysiology
• Erythrocytes can cross alveolar-capillary membrane with consequent
hemoptysis
• Capillary leak results in radiographic infiltrate and rales
• Hypoxemia results from alveolar filling
• Infection-related bronchospasm
• Increase in respiratory drive  respiratory alkalosis
• Intrapulmonary shunting
• Reduced lung volume and compliance
3. Which of the following is incorrect regarding
“atypical” pathogens causing community acquired
pneumonia? (HPIM 19th Ch153, p.804):

A. Atypical organisms include Chlamydia, Legionella, Mycoplasma and


anaerobes
B. Respiratory viruses are considered as atypical pathogens
C. Atypical organisms cannot be seen on Gram’s stain or cultured on
standard media
D. Atypical organisms should be treated with macrolides,
fluoroquinolones or tetracycline because they are intrinsically
resistant to all Beta lactam agents
3. Which of the following is incorrect regarding
“atypical” pathogens causing community acquired
pneumonia? (HPIM 19th Ch153, p.804):

A. Atypical organisms include Chlamydia, Legionella, Mycoplasma


and anaerobes
B. Respiratory viruses are considered as atypical pathogens
C. Atypical organisms cannot be seen on Gram’s stain or cultured on
standard media
D. Atypical organisms should be treated with macrolides,
fluoroquinolones or tetracycline because they are intrinsically
resistant to all Beta lactam agents
Pneumonia – Etiology
• Most common – Streptococcus pneumoniae
• Atypical pathogens: cannot be seen on Gram’s stain or cultured on
standard media; therapy with Macrolides, FQ or Tetracycline
• Mycoplasma
• Legionella
• Chlamydia
• Respiratory viruses
• Anaerobes: significant role when aspiration has occurred days to
weeks before; significant risk factor: gingivitis
4. Which of the following is NOT a risk
factor for Legionnaire’s disease (HPIM 19 th

Ch153, p.805):
A. Recent ship cruise
B. Severe renal disease
C. HIV infection
D. Recent hospitalization
4. Which of the following is NOT a risk
factor for Legionnaire’s disease (HPIM 19 th

Ch153, p.805):
A. Recent ship cruise
B. Severe renal disease
C. HIV infection
D. Recent hospitalization
Risk factors for Legionella infection
• Diabetes
• Hematologic malignancy
• Cancer
• Severe renal disease
• HIV infection
• Smoking
• Male gender
• Recent hotel stay or ship cruise
5. Which of the following is often necessary to
differentiate community acquired pneumonia from
other differentials (HPIM 19th Ch153, p.805):

A. Complete blood count


B. Chest radiography
C. Chest CT scan
D. Sputum studies
5. Which of the following is often necessary to
differentiate community acquired pneumonia from
other differentials (HPIM 19th Ch153, p.805):

A. Complete blood count


B. Chest radiography
C. Chest CT scan
D. Sputum studies
Pneumonia – Clinical Diagnosis
• Differentials
• Bronchitis
• Heart failure
• Pulmonary embolism
• Pneumonitis
• Chest radiography necessary to differentiate CAP from others
• Radiographic clues:
• pneumatoceles suggest infection with S. aureus
• upper-lobe cavitating lesion suggests tuberculosis
6. To be adequate for culture, a sputum specimen
obtained from a patient suspected to have pneumonia
must have the following findings (HPIM 19th Ch153,
p.806):
A. >25 neutrophils and >10 squamous epithelial cells / LPF
B. >25 neutrophils and <10 squamous epithelial cells / LPF
C. >25 neutrophils and >10 squamous epithelial cells / HPF
D. >25 neutrophils and <10 squamous epithelial cells / HPF
6. To be adequate for culture, a sputum specimen
obtained from a patient suspected to have pneumonia
must have the following findings (HPIM 19th Ch153,
p.806):
A. >25 neutrophils and >10 squamous epithelial cells / LPF
B. >25 neutrophils and <10 squamous epithelial cells / LPF
C. >25 neutrophils and >10 squamous epithelial cells / HPF
D. >25 neutrophils and <10 squamous epithelial cells / HPF
Sputum Examination
• main purpose of the sputum Gram’s stain is to ensure that a sample is
suitable for culture
• Sensitivity and specificity of the sputum Gram’s stain and culture are
highly variable
• Even in cases of proven bacteremic pneumococcal pneumonia, the
yield of positive cultures from sputum samples is ≤50%
• Except for CAP patients admitted to the ICU, no data exist to show
that treatment directed at a specific pathogen is statistically superior
to empirical therapy
7. Which of the following statements
regarding diagnostics in pneumonia is correct
(HPIM 19 Ch153, p.806):
th

A. Yield from blood cultures is moderate to high, especially when


blood is extracted prior to treatment
B. Blood cultures should be performed in all patients admitted for CAP,
especially for the high-risk patients (e.g. neutropenia, asplenia,
severe CAP)
C. Sensitivity and specificity of Legionella urine antigen test is low
D. PCR of nasopharyngeal swabs is the standard for diagnosis of
respiratory viral infection
7. Which of the following statements
regarding diagnostics in pneumonia is correct
(HPIM 19 Ch153, p.806):
th

A. Yield from blood cultures is moderate to high, especially when


blood is extracted prior to treatment
B. Blood cultures should be performed in all patients admitted for CAP,
especially for the high-risk patients (e.g. neutropenia, asplenia,
severe CAP)
C. Sensitivity and specificity of Legionella urine antigen test is low
D. PCR of nasopharyngeal swabs is the standard for diagnosis of
respiratory viral infection
Other diagnostics
• Blood cultures:
• Low yield (5-14%)
• No longer de rigueur for all hospitalized patients
• Recommended for high-risk patients: neutropenia secondary to pneumonia,
asplenia, complement deficiencies, chronic liver disease, or severe CAP
• Legionella urine antigen test:
• Only detects serogroup 1 (most common in US)
• High Sensitivity (90%), high specificity (99%)
• PCR:
• Standard for diagnosis of respiratory viral infection
• Markers of infection: CRP and procalcitonin increase in the presence
of bacterial infection
8. Who among the following patients with CAP
would probably benefit from ICU admission
according to a severity-of-illness score (HPIM 19th
Ch153, p.806):
A. 45/M, confused patient with respiratory rate of 40
B. 68/F, with serum creatinine of 1.5 mg/dl and blood pressure of
90/60
C. 65/F, confused patient with respiratory rate of 35
D. 64/M, urea >7mmol/L, BP of 80/40
8. Who among the following patients with CAP
would probably benefit from ICU admission
according to a severity-of-illness score (HPIM 19th
Ch153, p.806):
A. 45/M, confused patient with respiratory rate of 40
B. 68/F, with serum creatinine of 1.5 mg/dl and blood pressure of
90/60
C. 65/F, confused patient with respiratory rate of 35
D. 64/M, urea >7mmol/L, BP of 80/40
CURB-65 Criteria
• C – Confusion
• U – Urea >7 mmol/L
• R – Respiratory rate ≥30/min
• B – Blood pressure, systolic ≤90 mmHg or diastolic ≤60 mmHg
• 65 – Age ≥65 years
• Score of 2 – admit to hospital
• Score of 3 – may require admission to ICU
9. What is the most important risk factor for
antibiotic resistant pneumococcal infection
(HPIM 19 Ch153, p.807):
th

A. Older age
B. Recent hospitalization
C. Concomitant HIV infection
D. Use of specific antibiotic within 3 months
9. What is the most important risk factor for
antibiotic resistant pneumococcal infection
(HPIM 19 Ch153, p.807):
th

A. Older age
B. Recent hospitalization
C. Concomitant HIV infection
D. Use of specific antibiotic within 3 months
Risk factors for Penicillin Resistant
Pneumococcal infection
• Age <2 years or >65 years
• Attendance at day care centers
• Recent antimicrobial therapy – most important risk factor
• Recent hospitalization
• HIV infection
Mechanisms of Pneumococcal resistance to
Penicillin
• Direct DNA incorporation and remodeling resulting from contact with
closely related oral commensal bacteria
• Natural transformation
• Mutation of certain genes
Mechanisms of Pneumococcal resistance to
other antibiotics
• Beta-lactams – low affinity penicillin binding protein
• Macrolides – ribosomal methylation, efflux mechanism
• Fluoroquinolones – changes in target site – topoisomerases secondary
to gene mutation; efflux pump
10. If CA-MRSA is considered in CAP patients
admitted to the ICU, which antibiotic must be added to
the initial empirical regimen? (HPIM 19th, Ch153
p.808):
A. Colistin
B. Vancomycin
C. Clindamycin
D. Oxacillin
10. If CA-MRSA is considered in CAP patients
admitted to the ICU, which antibiotic must be added to
the initial empirical regimen? (HPIM 19th, Ch153
p.808):
A. Colistin
B. Vancomycin
C. Clindamycin
D. Oxacillin
Initial antibiotic management
• Initial therapy is usually empirical, designed to cover the
most likely pathogens
• Atypical pathogen coverage (Cephalosporin + Macrolide OR
Fluoroquinolone alone) has been consistently associated with a
significant reduction in mortality rates compared with β-lactam alone
• Fluoroquinolone-based regimen should be used for patients recently
given a macrolide (within the last 3 months), and vice versa
• For CAP patients admitted to the ICU, the risk of infection with P.
aeruginosa or CA-MRSA is increased.
11. 67/F diabetic was admitted to the hospital because of 3 days
duration of fever, cough and difficulty of breathing. On PE, she
was awake and conscious, BP 130/80, HR 104, RR 30, T 38.0.
Fine rales were noted at the right base. Lab exam showed WBC at
15,000. BUN 6 mmol/L, Creatinine was normal. Your initial
antibiotic treatment is(HPIM 19th, Ch153 p.808):
A. Oral azithromycin + oral co-amoxiclav
B. Ceftriaxone + oral clarithromycin
C. Carbapenem + clindamycin
D. Piperacillin-tazobactam + ciprofloxacin
11. 67/F diabetic was admitted to the hospital because of 3 days
duration of fever, cough and difficulty of breathing. On PE, she
was awake and conscious, BP 130/80, HR 104, RR 30, T 38.0.
Fine rales were noted at the right base. Lab exam showed WBC at
15,000. BUN 6 mmol/L, Creatinine was normal. Your initial
antibiotic treatment is(HPIM 19th, Ch153 p.808):
A. Oral azithromycin + oral co-amoxiclav
B. Ceftriaxone + oral clarithromycin
C. Carbapenem + clindamycin
D. Piperacillin-tazobactam + ciprofloxacin
CURB-65 Criteria
• C – Confusion
• U – Urea >7 mmol/L
• R – Respiratory rate ≥30/min
• B – Blood pressure, systolic ≤90 mmHg or diastolic ≤60 mmHg
• 65 – Age ≥65 years
• Score of 2 – admit to hospital
• Score of 3 – may require admission to ICU
12. Most obvious risk factor in ventilator-
associated pneumonia (HPIM 19 , Ch153
th

p.810):
A. Suction catheter
B. Ventilator
C. Endotracheal tube
D. Nasogastric tube
12. Most obvious risk factor in ventilator-
associated pneumonia (HPIM 19 , Ch153
th

p.810):
A. Suction catheter
B. Ventilator
C. Endotracheal tube
D. Nasogastric tube
Ventilator-Associated Pneumonia
• Non-MDR pathogens predominate if VAP develops in the first 5–7
days of the hospital stay.
• Frequency of diagnosis is not static but changes with the duration of
mechanical ventilation with the highest hazard ratio in the first 5 days
and a plateau in additional cases (1% per day) after ~2 weeks.
• In chronic ventilator units, purulent tracheobronchitis becomes a
significant issue, often interfering with efforts to wean patients off
mechanical ventilation
Ventilator-Associated Pneumonia
• Critical factors in the pathogenesis:
• colonization of the oropharynx with pathogenic microorganisms
• aspiration of these organisms from the oropharynx into the lower respiratory
tract
• compromise of the normal host defense mechanisms.
• most obvious risk factor is the endotracheal tube
• While the presence of an endotracheal tube may prevent large-volume
aspiration, micro-aspiration is actually exacerbated by secretions pooling
above the cuff
Ventilator-Associated Pneumonia
• Most important risk factors for pathogenic microorganisms in the
oropharynx:
• antibiotic selection pressure
• cross-infection from other infected/colonized patients or contaminated
equipment
• malnutrition
• Pathogens such as P. aeruginosa almost never cause infection in
patients without prior exposure to antibiotics.
13. What is the final step in the development
of VAP (HPIM 19 , Ch153 p.810):
th

A. Oropharyngeal colonization
B. Microaspiration
C. Overwhelming of host defenses
D. Multiplication of microorganism
13. What is the final step in the development
of VAP (HPIM 19 , Ch153 p.810):
th

A. Oropharyngeal colonization
B. Microaspiration
C. Overwhelming of host defenses
D. Multiplication of microorganism
Ventilator-Associated Pneumonia -
Pathogenesis
• Endotracheal tube insertion and suctioning can damage tracheal
mucosa, facilitating tracheal colonization
• Pathogenic bacteria can form a glycocalyx biofilm on the tube’s
surface that protects them from antibiotics and host defenses
• Bacteria can be dislodged during suctioning and can reinoculate the
trachea; glycocalyx can embolize to distal airways
Ventilator-Associated Pneumonia -
Pathogenesis
• Colony counts increase to high levels, sometimes days
before the development of clinical pneumonia, suggesting
that the final step in VAP development, independent of aspiration and
oropharyngeal colonization, is the overwhelming of host defenses.
• State of immunoparalysis/immunosuppression for critically ill patients
• Mechanism not clear
• Hyperglycemia affects neutrophil function
• Keeping the blood sugar close to normal with exogenous insulin decreases
risk of infection
• More frequent blood transfusions also adversely affect the immune response
Bronchial asthma
14. Which of the following is a major risk
factor to develop bronchial asthma (HPIM
19 , Ch309 p.1669):
th

A. History of allergic rhinitis


B. Infrequent infections during childhood
C. Obesity in a female patient
D. High concentration of air pollutants
14. Which of the following is a major risk
factor to develop bronchial asthma (HPIM
19 , Ch309 p.1669):
th

A. History of allergic rhinitis


B. Infrequent infections during childhood
C. Obesity in a female patient
D. High concentration of air pollutants
Asthma risk factors
• Atopy
• Major risk factor
• 80% of asthmatics may have allergic rhinitis
• Dietary factors that may lead to asthma:
• Low in: antioxidants, Vitamin A,C,D , Magnesium, Selenium, Omega 3
• High in: Sodium, Omega 6
• Obesity
• Independent risk factor, especially in women
15. Which of the following is the most common
trigger of acute severe exacerbation of bronchial
asthma (HPIM 19th, Ch309 p.1671):

A. Upper respiratory tract infection


B. Exercise during winter
C. Food allergy
D. Exposure to Dermatophagoides sp.
15. Which of the following is the most common
trigger of acute severe exacerbation of bronchial
asthma (HPIM 19th, Ch309 p.1671):

A. Upper respiratory tract infection


B. Exercise during winter
C. Food allergy
D. Exposure to Dermatophagoides sp.
Asthma triggers – Allergens
• Most common allergens – Dermatophagoides species
• Environmental exposure leads to low-grade chronic symptoms
• Perennial allergens – derived from cats, pets, cockroaches
• Seasonal allergens – grass pollen, fungal spores, thunderstorm
Asthma triggers – Viral infections
• Rhinovirus, RSV, coronavirus – most common triggers of SEVERE
asthma exacerbation
• Increase airway inflammation with increased numbers of eosinophils
and neutrophils
• Reduced type I interferons result in increased susceptibility to these
viral infections
Asthma triggers – Pharmacologic agents
• Beta-adrenergic blockers acutely worsen asthma – increased
cholinergic bronchoconstriction
• All Beta blockers need to be avoided (even selective B2 blockers or
topical application)
• ACE inhibitors inhibit breakdown of kinins (bronchoconstrictors), but
rarely worsen asthma
• Aspirin may worsen asthma in some patients
16. Which of the following is true regarding
Exercise Induced Asthma? (HPIM 19 , th

Ch309 p.1671):
A. Worse in hot, humid conditions
B. Typically begins during strenuous portion of exercise
C. Always requires treatment with bronchodilators
D. Best prevented by inhaled corticosteroids
16. Which of the following is true regarding
Exercise Induced Asthma? (HPIM 19 , th

Ch309 p.1671):
A. Worse in hot, humid conditions
B. Typically begins during strenuous portion of exercise
C. Always requires treatment with bronchodilators
D. Best prevented by inhaled corticosteroids
Asthma triggers – Exercise
• Exercise-induced asthma (EIA) typically begins after exercise has
ended and recovers spontaneously within about 30 minutes
• EIA is worse in cold, dry climates (cross-country running in cold
weather, overland skiing, ice hockey) than in hot, humid conditions
• May be prevented by prior administration of β2-agonists and
antileukotrienes, but BEST prevented by regular treatment with ICS
Asthma triggers – Physical factors
• Cold air and hyperventilation trigger asthma through same
mechanism as exercise
• Laughter
• Hot weather in some patients
• Weather changes
• Strong smells and perfumes
Asthma triggers – Food
• Little evidence that allergic reactions to food lead to increased asthma
symptoms
• Food preservative metabisulfite may trigger asthma through release
of sulfur dioxide gas in the stomach
Asthma triggers – GERD
• Common in asthma
• GERD increased by bronchodilators
• Acid reflux may trigger reflex bronchoconstriction, but rarely causes
asthma symptoms
• Anti-reflux therapy fails to reduce symptoms
Asthma triggers – Others
• Air pollution: Increased levels of sulfur dioxide, ozone, and nitrogen
oxides associated with increased symptoms
• Occupational:
• symptoms during working hours with relief on weekends and holidays
• If removed from exposure within first 6 months of symptoms, there is usually
complete recovery
Asthma triggers – Others
• Hormonal factors:
• Pre-menstrual worsening may be related to fall in progesterone and may be
improved by progesterone or gonadotropin-releasing factors
• Thyrotoxicosis & hypothyroidism can worsen asthma, mechanism is uncertain
• Stress:
• Worsens symptoms
• Severe stress usually does not worsen, and may even improve, asthma
symptoms
Asthma triggers – Air pollution
• Increased levels of sulfur dioxide, ozone, and nitrogen oxides
associated with increased symptoms
17. Which of the following is a common
pathologic finding in Fatal Asthma (HPIM
19 , Ch309 p.1672):
th

A. Uniform thinning of the airway wall


B. Occlusion of the airway by mucus plugs
C. Vasoconstriction of the bronchial vessels
D. Presence of exudates in the alveoli
17. Which of the following is a common
pathologic finding in Fatal Asthma (HPIM
19 , Ch309 p.1672):
th

A. Uniform thinning of the airway wall


B. Occlusion of the airway by mucus plugs
C. Vasoconstriction of the bronchial vessels
D. Presence of exudates in the alveoli
Bronchial asthma – Pathology
• Airway mucosa is infiltrated with activated eosinophils, T lymphocytes
and mast cells
• Degree of inflammation is poorly related to disease severity
• Inflammation is reduced by treatment with ICS
• Thickening of the basement membrane (also found in eosinophilic
bronchitis)
• Epithelium is often shed or friable, with reduced attachments to
airway wall
Bronchial asthma – Pathology
• Mucus plug – comprised of mucus and plasma proteins
• Vasodilation and increased numbers of blood vessels (angiogenesis)
• Pathology is uniform in different types of asthma (atopic, non-atopic,
occupational, aspirin-sensitive, pediatric)
• Changes do not extend to lung parenchyma
• Involvement of airways may be patchy, and this is consistent with
bronchographic findings of uneven narrowing of the airways.
18. Which of the following is the
characteristic physiologic abnormality of
bronchial asthma (HPIM 19 , Ch309 p.1675):
th

A. Air trapping
B. Increase in airway resistance
C. Airway hyperresponsiveness
D. Decrease in sympathetic activity in the airway
18. Which of the following is the
characteristic physiologic abnormality of
bronchial asthma (HPIM 19 , Ch309 p.1675):
th

A. Air trapping
B. Increase in airway resistance
C. Airway hyperresponsiveness
D. Decrease in sympathetic activity in the airway
Bronchial asthma – physiology
• Airway hyperresponsiveness
• Characteristic physiologic abnormality
• Excessive bronchoconstrictor response to inhaled triggers
• Limitation of airflow: due mainly to bronchoconstriction
• Results in reduction in forced expiratory volume in 1 second (FEV1),
FEV1/forced vital capacity (FVC) ratio, and peak expiratory flow (PEF)
19. Which of the following clinical features is
not consistent with bronchial asthma (HPIM
19 , Ch309 p.1675):
th

A. Symptoms worse during the day


B. Variable wheezing, dyspnea, and coughing
C. Increased mucus production with difficulty expectorating
D. Prodromal symptoms may precede an exacerbation
19. Which of the following clinical features is
not consistent with bronchial asthma (HPIM
19 , Ch309 p.1675):
th

A. Symptoms worse during the day


B. Variable wheezing, dyspnea, and coughing
C. Increased mucus production with difficulty expectorating
D. Prodromal symptoms may precede an exacerbation
Bronchial asthma – clinical features
• Characteristic symptoms:
• wheezing
• dyspnea
• coughing
• Symptoms typically worse at night
• Increased mucus production in some patients, with typically tenacious
mucus that is difficult to expectorate
• Prodromal symptoms may precede attack:
• itching under the chin, discomfort between the scapulae, or inexplicable fear
(impending doom)
20. A 28/M presents with occasional wheezing and
shortness of breath, triggered by inhalation of dust and
usually worse at night. Which of the following
confirms the diagnosis (GINA 2017):
A. Average daily diurnal PEF variability of 5-8%
B. Spirometry results showing obstructive pattern and an increase in
FEV1 of 15% and 160ml from baseline after inhaled Salbutamol
C. Fall in FEV1 of 12% from baseline after methacholine challenge test
D. Spirometry results showing obstructive pattern and an increase in
FEV1 of 15% and 220ml from baseline after 4 weeks of anti-
inflammatory treatment
20. A 28/M presents with occasional wheezing and
shortness of breath, triggered by inhalation of dust and
usually worse at night. Which of the following
confirms the diagnosis (GINA 2017):
A. Average daily diurnal PEF variability of 5-8%
B. Spirometry results showing obstructive pattern and an increase in
FEV1 of 15% and 160ml from baseline after inhaled Salbutamol
C. Fall in FEV1 of 12% from baseline after methacholine challenge test
D. Spirometry results showing obstructive pattern and an increase in
FEV1 of 15% and 220ml from baseline after 4 weeks of anti-
inflammatory treatment
Initial diagnosis of bronchial asthma
21. Which of the following is an incorrect
statement regarding Beta 2 agonists (HPIM
19th, Ch309 p.1676):
A. Treatment with beta agonists leads to increase in intracellular cAMP,
which relaxes smooth-muscle cells
B. Beta 2 agonists reverse and prevent contraction of airway smooth-
muscle cells by all known bronchoconstrictors
C. Long acting beta agonists may be given as a monotherapy for
patients with mild symptoms
D. Adverse effects are not usually a problem with beta 2 agonists when
given by inhalation
21. Which of the following is an incorrect
statement regarding Beta 2 agonists (HPIM
19th, Ch309 p.1676):
A. Treatment with beta agonists leads to increase in intracellular cAMP,
which relaxes smooth-muscle cells
B. Beta 2 agonists reverse and prevent contraction of airway smooth-
muscle cells by all known bronchoconstrictors
C. Long acting beta agonists may be given as a monotherapy for
patients with mild symptoms
D. Adverse effects are not usually a problem with beta 2 agonists when
given by inhalation
Beta agonists
• MOA: leads to increase in intracellular cAMP, which relaxes smooth-
muscle cells of all airways reversing and preventing contraction of
airway smooth-muscle cells by all known bronchoconstrictors
• SABAs (albuterol, terbutaline) have duration of action 3-6h
• LABAs (salmeterol, formoterol) have duration over 12h
• Should not be given without ICS therapy because they cannot control underlying
inflammation
• Slight excess in mortality associated with the use of LABA is related to the lack of
use of concomitant ICS
• Adverse effects not a problem during inhalation
• Most common: muscle tremor, palpitations
• Small decrease in plasma potassium
Anticholinergics
• MOA: prevent cholinergic nerve-induced bronchoconstriction and
mucus secretion
• Less effective than β2-agonists in asthma therapy because they inhibit
only the cholinergic reflex component of bronchoconstriction
• Additional bronchodilator when not controlled by ICS and LABA
combinations
• Slower onset of bronchodilatation compared to β2-agonists
• Adverse effects not a problem during inhalation
• Most common: dry mouth
• Others: urinary retention, glaucoma
22. Which of the following agents may decrease
steroid insensitivity in severe asthma by
activating histone deacetylase-2, thereby
switching off activated inflammatory genes
(HPIM 19th, Ch309 p.1677):
A. Beta agonists
B. Anticholinergics
C. Theophylline
D. Omalizumab
22. Which of the following agents may decrease
steroid insensitivity in severe asthma by
activating histone deacetylase-2, thereby
switching off activated inflammatory genes
(HPIM 19th, Ch309 p.1677):
A. Beta agonists
B. Anticholinergics
C. Theophylline
D. Omalizumab
Theophylline
• MOA: inhibition of phosphodiesterases in airway smooth-muscle cells,
which increases cyclic AMP
• Decreases steroid insensitivity in severe asthma by activating histone
deacetylase-2
• Adverse effects:
• Most common: nausea, vomiting, and headaches due to phosphodiesterase
inhibition
• Diuresis, palpitations, cardiac arrhythmias, epileptic seizures, and death may
occur due to adenosine A1-receptor antagonism.
23. 35/F consults for asthma. She was previously
on maintenance but stopped due to “feeling fine”.
She uses her Salbutamol inhaler 2-3x a week due
to shortness of breath. What is the appropriate
action to take (HPIM 19th, Ch309 p.1679):
A. Continue present treatment with Salbutamol as needed
B. Repeat spirometry prior to treatment
C. Restart maintenance therapy with inhaled corticosteroids
D. Discontinue Salbutamol as it may increase risk for mortality
23. 35/F consults for asthma. She was previously
on maintenance but stopped due to “feeling fine”.
She uses her Salbutamol inhaler 2-3x a week due
to shortness of breath. What is the appropriate
action to take (HPIM 19th, Ch309 p.1679):
A. Continue present treatment with Salbutamol as needed
B. Repeat spirometry prior to treatment
C. Restart maintenance therapy with inhaled corticosteroids
D. Discontinue Salbutamol as it may increase risk for mortality
Stepwise therapy
• Mild, intermittent asthma: SABA is enough
• Reliever >2x per week indicates need for controller
• Treatment of choice: ICS given twice daily
• Intermediate dose (e.g., 200 μg bid of beclomethasone dipropionate [BDP]) or
equivalent
• Decrease the dose if symptoms are controlled after 3 months
• Add LABA if symptoms not controlled
Inhaled corticosteroids
• Most effective controllers for asthma
• Reduces inflammatory cells, eosinophils, sputum and mast cells in the
airway
• Rapidly improve symptoms; improves lung function over days
• Reduces airway hyperresponsiveness
• Adverse effects:
• Local: hoarseness (dysphonia) and oral candidiasis
• Minimal systemic effects
24. 27/M consulting because of occasional shortness of
breath and wheezing 1-2x a month. On PE, clear breath
sounds. Chest x-ray was unremarkable and pulmonary
function test showed more than 15% improvement post-
bronchodilator in FEV1 and FEV1 %. Management for
this patient is (HPIM 19th, Ch309 p.1679):
A. ICS low-dose + β2 agonist as needed
B. LABA + ICS low-dose + β2 agonist as needed
C. Inhaled β2 agonist
D. Oral β2 agonist needed
24. 27/M consulting because of occasional shortness of
breath and wheezing 1-2x a month. On PE, clear breath
sounds. Chest x-ray was unremarkable and pulmonary
function test showed more than 15% improvement post-
bronchodilator in FEV1 and FEV1 %. Management for
this patient is (HPIM 19th, Ch309 p.1679):
A. ICS low-dose + β2 agonist as needed
B. LABA + ICS low-dose + β2 agonist as needed
C. Inhaled β2 agonist
D. Oral β2 agonist needed
Diagnosis: Mild Intermittent Asthma
25. A 28/M on maintenance inhalers presents with night
waking due to difficulty of breathing once a week. He
experiences symptoms daily at work and claims to use
his rescue inhaler 3x a week. He denies limitation of
activity. What is the level of control of his BA (GINA
2017):
A. Uncontrolled
B. Partly controlled
C. Moderately controlled
D. Well controlled
25. A 28/M on maintenance inhalers presents with night
waking due to difficulty of breathing once a week. He
experiences symptoms daily at work and claims to use
his rescue inhaler 3x a week. He denies limitation of
activity. What is the level of control of his BA (GINA
2017):
A. Uncontrolled
B. Partly controlled
C. Moderately controlled
D. Well controlled
Control of bronchial asthma
26. What is the most common reason for poor
control of asthma (HPIM 19 , Ch309
th

p.1679):
A. Steroid resistance
B. Gastroesophageal reflux disease
C. Concomitant use of other medication
D. Non-compliance with medication
26. What is the most common reason for poor
control of asthma (HPIM 19 , Ch309
th

p.1679):
A. Steroid resistance
B. Gastroesophageal reflux disease
C. Concomitant use of other medication
D. Non-compliance with medication
Mechanisms of Poor Control
• Non-compliance with medication – most common reason
• No immediate benefit felt/ concern for adverse events
• Measure through fractional excretion of Nitric Oxide
• Improved by combination with LABA
• Exposure to high levels of allergens and occupational agents
• Severe rhinosinusitis
• Upper airway disease
Mechanisms of Poor Control
• Drugs – ASA, Beta-blockers, COX-inhibitors
• Pre-menstruation – treat with progesterone or gonadotropin-
releasing factors
• Hypothyroidism and hyperthyroidism
Refractory asthma
• 5-10% of asthmatics
• 2 major patterns
1. Persistent symptoms and poor lung function
2. Near normal lung function but intermittent severe exacerbations
• Treatment:
• Maintenance with oral steroids
• Low doses of theophylline
• Omalizumab for allergic asthma
• Anti TNF should NOT be used
Cortico-steroid resistant asthma
• Defined by a failure to respond to a high dose of oral prednisone/
prednisolone (40 mg once daily over 2 weeks), ideally with a 2-week
run-in with matched placebo
• More common is reduced responsiveness to corticosteroids where
control of asthma requires OCS (corticosteroid dependent asthma).
• In patients with poor responsiveness to corticosteroids, there is a
reduction in the response of circulating monocytes and lymphocytes
to the anti-inflammatory effects of corticosteroids in vitro and
reduced skin blanching in response to topical corticosteroids.
Brittle asthma
• Type 1: persistent pattern of variability and may require oral
corticosteroids or, at times, continuous infusion of β2-agonists
• Type 2: others have generally normal or near-normal lung function
but precipitous, unpredictable falls in lung function that may result in
death
• difficult to manage because they do not respond well to corticosteroids, and
the worsening of asthma does not reverse well with inhaled bronchodilators.
• The most effective therapy is subcutaneous epinephrine, which suggests that
the worsening is likely to be a localized airway anaphylactic reaction with
edema.
• In some of these patients, there may be allergy to specific foods.
27. Which of the following is TRUE
regarding Aspirin-Sensitive Asthma? (HPIM
19 , Ch309 p.1680):
th

A. Asthma is usually followed by perennial rhinitis and nasal polyps


B. Rhinorrhea, conjunctival injection and wheezing are brought about
only by large doses of Aspirin
C. All nonselective and selective COX inhibitors should be avoided
D. There is a genetic predisposition to increased production of
cysteinyl-leukotrienes with functional polymorphism of cys-
leukotriene C4 synthase
27. Which of the following is TRUE
regarding Aspirin-Sensitive Asthma? (HPIM
19 , Ch309 p.1680):
th

A. Asthma is usually followed by perennial rhinitis and nasal polyps


B. Rhinorrhea, conjunctival injection and wheezing are brought about
only by large doses of Aspirin
C. All nonselective and selective COX inhibitors should be avoided
D. There is a genetic predisposition to increased production of
cysteinyl-leukotrienes with functional polymorphism of cys-
leukotriene C4 synthase
Aspirin-sensitive asthma
• 1–5% of asthmatics become worse with aspirin and other COX
inhibitors (more commonly in severe cases and with frequent hospital
admission)
• usually preceded by perennial rhinitis and nasal polyps in non-atopic
patients with a late onset of the disease
• Aspirin, even in small doses, characteristically provokes rhinorrhea,
conjunctival injection, facial flushing, and wheezing.
Aspirin-sensitive asthma
• There is a genetic predisposition to increased production of cysteinyl-
leukotrienes with functional polymorphism of cys-leukotriene C4
synthase.
• Asthma is triggered by COX inhibitors but is persistent even in their
absence. All nonselective COX inhibitors should be avoided, but
selective COX2 inhibitors are safe to use.
• Aspirin-sensitive asthma responds to usual therapy with ICS. Although
antileukotrienes should be effective in these patients, they are no
more effective than in allergic asthma
28. Which of the following has been shown to be
safe for use in pregnancy and without teratogenic
potential? (HPIM 19th, Ch309 p.1680):

A. LABA
B. Oral corticosteroids
C. Omalizumab
D. Theophylline
28. Which of the following has been shown to be
safe for use in pregnancy and without teratogenic
potential? (HPIM 19th, Ch309 p.1680):

A. LABA
B. Oral corticosteroids
C. Omalizumab
D. Theophylline
Asthma in pregnancy
• Rule of thirds: 1/3 improve, 1/3 worsen, 1/3 same
• Poor control may have adverse effects on fetal development
• Shown to be safe and without teratogenic potential: [SIT]
• SABA, ICS, Theophylline
• Less safety information about newer classes of drugs such as LABA,
antileukotrienes, and anti-IgE
• Better to use prednisone rather than prednisolone because it cannot
be converted to the active prednisolone by the fetal liver
• Breast-feeding is not contraindicated while using these medications
COPD
29. What is the most typical finding in
COPD? (HPIM 19 , Ch314 p. 1701)
th

A. Persistent reduction in forced expiratory flow rate


B. Increase in residual volume
C. Increase in total lung capacity
D. Presence of ventilation-perfusion mismatch
29. What is the most typical finding in
COPD? (HPIM 19 , Ch314 p. 1701)
th

A. Persistent reduction in forced expiratory flow rate


B. Increase in residual volume
C. Increase in total lung capacity
D. Presence of ventilation-perfusion mismatch
COPD – Pathophysiology
• Airflow obstruction
• Chronic reduction in FEV1/FVC
• Reduced FEV1 does not show large responses to inhaled bronchodilators
• Less elastic recoil
• Fall in cross-sectional area of the airways
• Hyperinflation
• Air trapping (increased residual volume)
• Increased total lung capacity
• Push diaphragm into flattened position
30. Which of the following is an expected
finding in gas exchange in patients with
COPD? (HPIM 19 , Ch314 p. 1702)
th

A. PaO2 usually remains near normal until the FEV1 is decreased to


25% of predicted
B. PaCO2 starts to elevate when FEV1 is decreased to <25% of
predicted
C. Uniform ventilation-perfusion mismatching is found in most
patients
D. Intrapulmonary shunting is the main cause of hypoxemia in COPD
30. Which of the following is an expected
finding in gas exchange in patients with
COPD? (HPIM 19 , Ch314 p. 1702)
th

A. PaO2 usually remains near normal until the FEV1 is decreased to


25% of predicted
B. PaCO2 starts to elevate when FEV1 is decreased to <25% of
predicted
C. Uniform ventilation-perfusion mismatching is found in most
patients
D. Intrapulmonary shunting is the main cause of hypoxemia in COPD
COPD – Gas Exchange
• The PaO2 usually remains near normal until the FEV1 is decreased to
50% of predicted
• Elevation of arterial level of CO2 (PaCO2) is not expected until FEV1 <
25% of predicted
• Pulmonary hypertension severe enough to cause cor pulmonale and
RV failure due to COPD typically when FEV1 < 25% of predicted and
with chronic hypoxemia (PaO2 < 55 mm Hg)
31. The predominant gas exchange problem
that accounts for reduction in PaO2 in COPD?
(HPIM 19th, Ch314 p. 1702)
A. Shunting
B. Ventilation-perfusion mismatch
C. Air trapping
D. Low inspired FiO2
31. The predominant gas exchange problem
that accounts for reduction in PaO2 in COPD?
(HPIM 19th, Ch314 p. 1702)
A. Shunting
B. Ventilation-perfusion mismatch
C. Air trapping
D. Low inspired FiO2
COPD – Gas Exchange
• Non-uniform ventilation and V/Q mismatch
• Regional differences in compliance and airway resistance
• Shunting is minimal
• Decreased PaO2 due to V/Q mismatch
• Modest increase in FiO2 is effective in treating hypoxemia
32. What is the most highly significant
predictor of FEV1 in COPD? (HPIM 19th,
Ch314 p. 1702)
A. Environmental factors
B. Genetic factors
C. Pack-years of cigarette smoking
D. Age
32. What is the most highly significant
predictor of FEV1 in COPD? (HPIM 19th,
Ch314 p. 1702)
A. Environmental factors
B. Genetic factors
C. Pack-years of cigarette smoking
D. Age
33. Which of the following is true regarding
the pathology of COPD? (HPIM 19 , Ch314
th

p. 1702)
A. Major site of resistance are in airways ≤ 2mm in diameter
B. Neutrophils accumulate and are 5x as many in respiratory
bronchioles
C. Panacinar emphysema is most frequently associated with cigarette
smoking
D. Alpha1 antitrypsin deficiency has predilection for upper lobes
33. Which of the following is true regarding
the pathology of COPD? (HPIM 19 , Ch314
th

p. 1702)
A. Major site of resistance are in airways ≤ 2mm in diameter
B. Neutrophils accumulate and are 5x as many in respiratory
bronchioles
C. Panacinar emphysema is most frequently associated with cigarette
smoking
D. Alpha1 antitrypsin deficiency has predilection for upper lobes
COPD – Pathology
• Cigarette smoke exposure may affect large airways, small airways (≤ 2
mm diameter) and alveoli
• Changes in large airways cause cough and sputum
• Changes in small airways and alveoli cause physiologic alterations
• ≤ 2 mm diameter airways are the site of increased resistance
• Findings:
• Smooth muscle hypertrophy
• Luminal narrowing by fibrosis, excess mucus, edema and cellular infiltration
• Reduced surfactant increases surface tension
COPD – Pathology
• Centriacinar emphysema
• Associated with smoking
• Enlarged airspaces in association with respiratory bronchioles
• Upper lobes and superior segments of lower lobes
• Panacinar emphysema
• Abnormality of large air spaces evenly distributed within and across acinar
units
• Usually observed in α1AT deficiency, which has a predilection for the lower
lobes
COPD – Pathology
• Bronchoalveolar lavage fluid from such individuals contains roughly
five times as many macrophages as lavage from nonsmokers.
• In smokers’ lavage fluid, macrophages comprise >95% of the total cell
count, and neutrophils, nearly absent in nonsmokers’ lavage, account
for 1–2% of the cells.
34. 52/M heavy smoker for 40 years, presents with
chronic cough, easy fatigability and production of thick,
copious sputum. What is the appropriate step to take in
the management of this patient? (GOLD 2017)
A. Order pulmonary function test to confirm the diagnosis
B. Begin treatment since he presents with the classic symptoms and
can be diagnosed clinically
C. Order chest CT scan with IV contrast to rule out lung CA
D. Use validated scoring tool to determine severity of his illness
34. 52/M heavy smoker for 40 years, presents with
chronic cough, easy fatigability and production of thick,
copious sputum. What is the appropriate step to take in
the management of this patient? (GOLD 2017)
A. Order pulmonary function test to confirm the diagnosis
B. Begin treatment since he presents with the classic symptoms and
can be diagnosed clinically
C. Order chest CT scan with IV contrast to rule out lung CA
D. Use validated scoring tool to determine severity of his illness
35. What is the current definitive test for establishing
the presence or absence of emphysema in living
subjects? (HPIM 19th, Ch314 p. 1705)

A. CT scan
B. Chest X-ray
C. Spirometry
D. MRI
35. What is the current definitive test for establishing
the presence or absence of emphysema in living
subjects? (HPIM 19th, Ch314 p. 1705)

A. CT scan
B. Chest X-ray
C. Spirometry
D. MRI
COPD - Imaging
• Suggestive of emphysema and hyperinflation:
• obvious bullae
• paucity of parenchymal markings
• hyperlucency
• increased lung volumes
• flattening of the diaphragm
• Computed tomography (CT) scan is the current definitive test for
establishing the presence or absence of emphysema
• From a practical perspective, the CT scan currently does little to
influence therapy of COPD except in individuals considering surgical
therapy for their disease and as screening for lung cancer.
36. Which of the following drugs can be used
for smoking cessation? (HPIM 19 , Ch314 p.
th

1705)
A. Electronic cigarettes
B. Bupropion
C. Nitrate patch
D. Carbamazepine
36. Which of the following drugs can be used
for smoking cessation? (HPIM 19 , Ch314 p.
th

1705)
A. Electronic cigarettes
B. Bupropion
C. Nitrate patch
D. Carbamazepine
Smoking cessation
• Middle-aged smokers who were able to successfully stop smoking
experienced a significant improvement in the rate of decline in
pulmonary function
• ALL adult, non-pregnant smokers considering quitting be offered
pharmacotherapy, in the absence of any contraindication to
treatment
• Three principal pharmacologic approaches:
• Bupropion (anti-depressant)
• Nicotine replacement therapy (gum, patch, lozenge, inhaler, nasal spray)
• Varenicline (nicotinic acid receptor agonist/antagonist)
37. What intervention has been demonstrated
to influence the natural history of patients
with COPD?(HPIM 19 , Ch314 p. 1705)
th

A. Oxygen therapy
B. β2 agonists
C. Pulmonary rehab
D. Pneumococcal vaccine
37. What intervention has been demonstrated
to influence the natural history of patients
with COPD?(HPIM 19 , Ch314 p. 1705)
th

A. Oxygen therapy
B. β2 agonists
C. Pulmonary rehab
D. Pneumococcal vaccine
COPD – Natural history
• Three interventions have been demonstrated to affect natural history
of patients with COPD
• Smoking cessation
• O2 therapy in chronically hypoxemic patients
• Lung volume reduction surgery in selected patients with emphysema
• There is currently suggestive, but not definitive evidence, that the use
of inhaled glucocorticoids may alter mortality rate (but not lung
function)
• Other therapies are directed at improving symptoms and decreasing
frequency and severity of exacerbations
38. Which group of patients with stable
COPD would benefit the most from oxygen
therapy? (HPIM 19 , Ch314 p. 1705)
th

A. Patients with resting O2 saturation <92%


B. Patients with resting O2 saturation <90% with concomitant
obstructive sleep apnea
C. Patients with resting O2 saturation <90% with signs of cor
pulmonale
D. Patients with resting CO2 > 60mmHg
38. Which group of patients with stable
COPD would benefit the most from oxygen
therapy? (HPIM 19 , Ch314 p. 1705)
th

A. Patients with resting O2 saturation <92%


B. Patients with resting O2 saturation <90% with concomitant
obstructive sleep apnea
C. Patients with resting O2 saturation <90% with signs of cor
pulmonale
D. Patients with resting CO2 > 60mmHg
COPD – Oxygen therapy
• The use of O2 has been demonstrated to have a significant impact on
mortality rate in:
• Resting O2 saturation ≤ 88%
• Resting O2 saturation ≤ 90% with signs of pulmonary hypertension or right
heart failure
• Patients meeting the criteria should be on continued oxygen
supplementation as its mortality benefit is proportional to the
number of hours/day oxygen is used
• Supplemental O2 is commonly prescribed in patients with exertional
hypoxemia or nocturnal hypoxemia  benefits not yet well-
substantiated
39. The following are bacteria frequently
implicated in COPD exacerbations EXCEPT?
(HPIM 19 , Ch314 p. 1707)
th

A. Haemophilus influenzae
B. Legionella pneumophila
C. Streptococcous pneumoniae
D. Moraxella catarrhalis
39. The following are bacteria frequently
implicated in COPD exacerbations EXCEPT?
(HPIM 19 , Ch314 p. 1707)
th

A. Haemophilus influenzae
B. Legionella pneumophila
C. Streptococcous pneumoniae
D. Moraxella catarrhalis
COPD – Bacteria implicated in exacerbations
• Bacteria frequently implicated in COPD exacerbations:
• Streptococcus pneumonia
• Haemophilus influenza
• Moraxella catarrhalis
• In addition, Mycoplasma pneumoniae or Chlamydia pneumoniae are
found in 5–10% of exacerbations.
40. 72/M, recently diagnosed COPD through PFT after
presenting with shortness of breath. There is no history of
hospital admission for exacerbation. What is the
appropriate maintenance therapy for him? (GOLD 2017)

A. LAMA alone
B. LAMA + LABA
C. LAMA + LABA + ICS
D. LAMA + LABA + Roflumilast
40. 72/M, recently diagnosed COPD through PFT after
presenting with shortness of breath. There is no history of
hospital admission for exacerbation. What is the
appropriate maintenance therapy for him? (GOLD 2017)

A. LAMA alone
B. LAMA + LABA
C. LAMA + LABA + ICS
D. LAMA + LABA + Roflumilast
41. What is a known benefit of glucocorticoid use
in COPD patients admitted to the hospital due to
exacerbation? (HPIM 19th, Ch314 p. 1707)

A. Shortens hospital stay


B. Hasten recovery
C. Reduce chance of subsequent exacerbation or relapse
D. All of the above
41. What is a known benefit of glucocorticoid use
in COPD patients admitted to the hospital due to
exacerbation? (HPIM 19th, Ch314 p. 1707)

A. Shortens hospital stay


B. Hasten recovery
C. Reduce chance of subsequent exacerbation or relapse
D. All of the above
Glucocorticoids in COPD
• GOLD guidelines recommend:
• 30–40 mg of oral prednisolone or its equivalent for a period of 10–14 days.
• Hyperglycemia, particularly in patients with preexisting diagnosis of
diabetes, is the most frequently reported acute complication of
glucocorticoid treatment.
Pleural disorders
42. Which of the following is a mechanism of
pleural fluid accumulation? (HPIM 19 ,
th

Ch316 p. 1716)
A. Fluid is removed via lymphatics situated in the visceral pleura
B. Fluid does not enter pleural space from its interstitial spaces of the
lung
C. Fluid enters pleural space from capillaries in parietal pleura
D. Fluid cannot enter from the peritoneal cavity.
42. Which of the following is a mechanism of
pleural fluid accumulation? (HPIM 19 ,
th

Ch316 p. 1716)
A. Fluid is removed via lymphatics situated in the visceral pleura
B. Fluid does not enter pleural space from its interstitial spaces of the
lung
C. Fluid enters pleural space from capillaries in parietal pleura
D. Fluid cannot enter from the peritoneal cavity.
Etiology of pleural effusion
• Pleural effusion may develop when there is excess pleural fluid
formation or when there is decreased fluid removal by the
lymphatics.
• Fluid enters the pleural space from the capillaries in the parietal
pleura and is removed via the lymphatics in the parietal pleura.
• Fluid also can enter the pleural space from the interstitial spaces of
the lung via the visceral pleura or from the peritoneal cavity via small
holes in the diaphragm.
43. Which of the following is the best chest imaging
procedure to evaluate patients suspected to have
pleural effusion? (HPIM 19th, Ch316 p. 1716)

A. Chest xray
B. Lateral decubitus x-ray
C. Chest ultrasound
D. Chest CT scan
43. Which of the following is the best chest imaging
procedure to evaluate patients suspected to have
pleural effusion? (HPIM 19th, Ch316 p. 1716)

A. Chest xray
B. Lateral decubitus x-ray
C. Chest ultrasound
D. Chest CT scan
Imaging
• Chest ultrasound has replaced the lateral decubitus x-ray in the
evaluation of suspected pleural effusions and as a guide to
thoracentesis
44. The first step in evaluating pleural effusion is to
determine whether it is a transudate or an exudate.
Which of the following is consistent with an exudative
effusion? (HPIM 19th, Ch316 p. 1716)
A. Pleural fluid protein/serum protein >0.3
B. Pleural fluid protein/serum protein more than 2/3 the normal upper
limit
C. Pleural fluid LDH/serum LDH >0.5
D. Pleural fluid LDH/serum LDH >0.6
44. The first step in evaluating pleural effusion is to
determine whether it is a transudate or an exudate.
Which of the following is consistent with an exudative
effusion? (HPIM 19th, Ch316 p. 1716)
A. Pleural fluid protein/serum protein >0.3
B. Pleural fluid protein/serum protein more than 2/3 the normal upper
limit
C. Pleural fluid LDH/serum LDH >0.5
D. Pleural fluid LDH/serum LDH >0.6
Diagnostic approach
• Exudative pleural effusions meet at least one of the ff criteria:
• 1. Pleural fluid protein/serum protein >0.5
• 2. Pleural fluid LDH/serum LDH >0.6
• 3. Pleural fluid LDH more than two-thirds the normal upper limit for serum
• If one or more of the exudative criteria are met and the patient is
clinically thought to have a condition producing a transudative
effusion, compute for the difference between the protein levels in the
serum and the pleural fluid. If this gradient is >31 g/L (3.1 g/dL), it is
considered transudative.
45. When is thoracentesis indicated in a patient
suspected to have pleural effusion secondary to
heart failure? (HPIM 19th, Ch316 p. 1717)

A. Effusion is bilateral, comparable in size


B. Patient is afebrile
C. Patient has pleuritic chest pain
D. Thoracentesis is not indicated in this group of patients
45. When is thoracentesis indicated in a patient
suspected to have pleural effusion secondary to
heart failure? (HPIM 19th, Ch316 p. 1717)

A. Effusion is bilateral, comparable in size


B. Patient is afebrile
C. Patient has pleuritic chest pain
D. Thoracentesis is not indicated in this group of patients
Effusion due to heart failure
• Most common cause of pleural effusion: LV failure
• Thoracentesis should be performed if:
• Effusion is NOT bilateral, NOT comparable in size
• Presence of fever
• Presence of pleuritic chest pain (to verify that effusion is transudative)
• Effusion persists despite therapy
• Pleural fluid N-terminal proBNP > 1500 pg/ml virtually diagnostic of
effusion due to congestive heart failure
46. In patients with parapneumonic effusion who
underwent thoracentesis, which of the following
features would warrant a more invasive procedure?
(HPIM 19th, Ch316 p. 1717)
A. Presence of a loculated fluid
B. Pleural fluid pH of 7.50
C. Pleural fluid glucose of 70mg/dl
D. Pleural fluid microscopy demonstrating PMN > 25 / LPF
46. In patients with parapneumonic effusion who
underwent thoracentesis, which of the following
features would warrant a more invasive procedure?
(HPIM 19th, Ch316 p. 1717)
A. Presence of a loculated fluid
B. Pleural fluid pH of 7.50
C. Pleural fluid glucose of 70mg/dl
D. Pleural fluid microscopy demonstrating PMN > 25 / LPF
Parapneumonic effusion
• Most common cause of exudative effusion in USA
• If the free fluid separates the lung from the chest wall by >10 mm, a
therapeutic thoracentesis should be performed
• Factors indicating the likely need for a procedure more invasive than a
thoracentesis (in increasing order of importance):
• 1. Loculated pleural fluid
• 2. Pleural fluid pH <7.20
• 3. Pleural fluid glucose <3.3 mmol/L (<60 mg/dL)
• 4. Positive Gram stain or culture of the pleural fluid
• 5. Presence of gross pus in the pleural space
47. What can cause an increase in size of pleural
effusion secondary to pulmonary embolism after
anticoagulation? (HPIM 19th, Ch316 p. 1717)

A. Recurrence of embolism
B. Hemothorax
C. Pleural infection
D. All of the above
47. What can cause an increase in size of pleural
effusion secondary to pulmonary embolism after
anticoagulation? (HPIM 19th, Ch316 p. 1717)

A. Recurrence of embolism
B. Hemothorax
C. Pleural infection
D. All of the above
Effusion secondary to pulmonary embolism
• Diagnosis most commonly overlooked in a patient with an
undiagnosed pleural effusion: pulmonary embolism
• Diagnosis: spiral CT scan or pulmonary arteriography
• Treat as pulmonary embolism
• Increase in size after anticoagulation:
• Recurrent emboli
• Hemothorax
• Pleural infection
48. Which of the following is true regarding
tuberculous pleural effusion? (HPIM 19 ,th

Ch316 p. 1718)
A. Fluid is an exudate predominantly filled with neutrophils
B. Diagnosis may be established by demonstrating high levels of
interferon alpha or lactate dehydrogenase in the pleural fluid
C. Diagnosis can be established by culture of the pleural fluid or
needle biopsy
D. Treatment of tuberculous effusion requires a longer period than the
standard regimen for pulmonary tuberculosis
48. Which of the following is true regarding
tuberculous pleural effusion? (HPIM 19 ,th

Ch316 p. 1718)
A. Fluid is an exudate predominantly filled with neutrophils
B. Diagnosis may be established by demonstrating high levels of
interferon alpha or lactate dehydrogenase in the pleural fluid
C. Diagnosis can be established by culture of the pleural fluid or
needle biopsy
D. Treatment of tuberculous effusion requires a longer period than the
standard regimen for pulmonary tuberculosis
Tuberculous pleural effusion
• Associated with primary TB and are thought to be due primarily to a
hypersensitivity reaction to tuberculous protein in the pleural space
• Fluid is an exudate with predominantly small lymphocytes
• Diagnosis established:
• High levels of TB markers in the pleural fluid (adenosine deaminase >40 IU/L
or interferon γ >140 pg/mL)
• culture of the pleural fluid
• needle biopsy of the pleura
• Thoracoscopy
• Treatment same as pulmonary TB
Venous thromboembolism
49. Which of the following is a pathophysiologic
abnormality found in patients with PE ?(HPIM
19th, Ch300 p. 1631)

A. Increased alveolar dead space


B. Decreased Alveolar ventilation
C. Decreased physiologic dead space
D. Increased pulmonary compliance
49. Which of the following is a pathophysiologic
abnormality found in patients with PE ?(HPIM
19th, Ch300 p. 1631)

A. Increased alveolar dead space


B. Decreased Alveolar ventilation
C. Decreased physiologic dead space
D. Increased pulmonary compliance
Pathophysiology of pulmonary embolism
• Increased pulmonary vascular resistance:
• Vascular obstruction or platelet secretion of vasoconstricting
neurohumoral agents such as serotonin
• Vasoactive mediators can produce V/Q mismatch at sites remote
from embolus, thus accounting for discordance between small
embolus and large alveolar-arterial O2 gradient
Pathophysiology of pulmonary embolism
• Impaired gas exchange:
• Increased alveolar dead space from vascular obstruction
• Hypoxemia from alveolar hypoventilation relative to perfusion in
the non-obstructed lung
• Right-to-left shunting
• Impaired CO transfer due to loss of gas exchange surface
Pathophysiology of pulmonary embolism
• Alveolar hyperventilation due to reflex stimulation of irritant
receptors
• Increased airway resistance due to constriction of airways distal to the
bronchi
• Decreased pulmonary compliance due to lung edema, lung
hemorrhage, or loss of surfactant
50. Which of the following is an expected
clinical feature in patients with pulmonary
embolism?(HPIM 19 , Ch300 p. 1632)
th

A. Moderate to large PE causes RV hypokinesia on 2D Echo but


elevated systemic arterial pressure initially
B. Underfilling of LV may lead to a fall in cardiac output and systemic
arterial pressure
C. Large PE frequently causes pleuritic chest pain
D. Small to moderate PE produce depressed right heart function
50. Which of the following is an expected
clinical feature in patients with pulmonary
embolism?(HPIM 19 , Ch300 p. 1632)
th

A. Moderate to large PE causes RV hypokinesia on 2D Echo but


elevated systemic arterial pressure initially
B. Underfilling of LV may lead to a fall in cardiac output and systemic
arterial pressure
C. Large PE frequently causes pleuritic chest pain
D. Small to moderate PE produce depressed right heart function
Features of pulmonary embolism
• Increased pulmonary vascular resistance
• RV wall tension rises and causes further RV dilation and dysfunction
• RV contraction continues even after LV starts relaxing at end-systole
• IV septum bulges into and compresses normal LV
• Diastolic LV impairment develops resulting in reduced LV distensibility
and LV filling during diastole
Features of pulmonary embolism
• Increased RV wall tension
• Compresses right coronary artery
• Diminishes subendocardial perfusion
• Limits myocardial oxygen supply
• May precipitate myocardial ischemia and RV infarction
• Underfilling of LV
• Fall in LV cardiac output and systemic arterial pressure
• Compromised coronary artery perfusion
• Circulatory collapse and death may ensue
51. Which of the following is true regarding
the classification of PE? (HPIM 19 , Ch300
th

p. 1632)
A. Massive PE is characterized by extensive thrombosis affecting less
than half of the pulmonary vasculature
B. Massive PE may present as cardiogenic shock
C. Submassive PE is characterized by RV dysfunction and hypotension
D. Low risk PE has poor prognosis
51. Which of the following is true regarding
the classification of PE? (HPIM 19 , Ch300
th

p. 1632)
A. Massive PE is characterized by extensive thrombosis affecting less
than half of the pulmonary vasculature
B. Massive PE may present as cardiogenic shock
C. Submassive PE is characterized by RV dysfunction and hypotension
D. Low risk PE has poor prognosis
Classification of pulmonary embolism
• Massive PE (5-10%)
• Extensive thrombosis > 1/2 of pulmonary vasculature
• Dyspnea, syncope, hypotension, cyanosis are hallmarks
• May present in cardiogenic shock
• Submassive PE (20-25%)
• RV dysfunction + normal systemic arterial pressure
• Right heart failure + increase in cardiac biomarkers  increased risk for
clinical deterioration
• Low-risk PE (70-75%)
• Excellent prognosis
52. Which of the following features favor a
diagnosis of DVT? (HPIM 19 , Ch300 p.
th

1632)
A. A cramp in the lower calf persists and intensifies over several days
B. Severe calf discomfort occurs acutely
C. The affected leg is diffusely edematous and warm
D. Fever and chills are likely present during diagnosis
52. Which of the following features favor a
diagnosis of DVT? (HPIM 19 , Ch300 p.
th

1632)
A. A cramp in the lower calf persists and intensifies over several days
B. Severe calf discomfort occurs acutely
C. The affected leg is diffusely edematous and warm
D. Fever and chills are likely present during diagnosis
Clinical pearls of DVT
• Most common symptom: cramp or “charley horse” in the lower calf
that persists and intensifies over several days
• PE: mild palpation discomfort in the lower calf
• Massive DVT: marked thigh swelling, tenderness, erythema
• Differentials:
• Ruptured Baker’s cyst: sudden, severe calf discomfort
• Cellulitis: fever and chills
• Postthrombotic syndrome: diffuse edema of the leg
53. Which of the following is true regarding
D-dimer assay for VTE? (HPIM 19 , Ch300
th

p. 1632)
A. D-dimer assay has higher sensitivity in DVT rather than PE
B. D-dimer assay levels increase during 1st trimester of pregnancy
C. Patients with high clinical likelihood of VTE should skip D-dimer
testing
D. Elevation of D-dimer indicates clinically effective endogenous
thrombolysis
53. Which of the following is true regarding
D-dimer assay for VTE? (HPIM 19 , Ch300
th

p. 1632)
A. D-dimer assay has higher sensitivity in DVT rather than PE
B. D-dimer assay levels increase during 1st trimester of pregnancy
C. Patients with high clinical likelihood of VTE should skip D-dimer
testing
D. Elevation of D-dimer indicates clinically effective endogenous
thrombolysis
D-Dimer in VTE
• Elevation indicates clinically INEFFECTIVE thrombolysis
• More sensitive for PE than DVT because thrombus size is larger
• Levels increase in:
• Myocardial infarction
• Pneumonia
• Sepsis
• Cancer
• Postoperative state
• Second or third trimester of pregnancy
• Patients with high clinical likelihood of VTE should skip D-dimer
testing
54. You are suspecting PE in a 68/M, bed-ridden
with prostate CA and tenderness in the calf. What
is the appropriate diagnostic test to establish
diagnosis? (HPIM 19th, Ch300 p. 1632)
A. D-Dimer assay
B. 2D Echo with Doppler studies
C. Venous ultrasound of the calf
D. Chest CT with IV contrast
54. You are suspecting PE in a 68/M, bed-ridden
with prostate CA and tenderness in the calf. What
is the appropriate diagnostic test to establish
diagnosis? (HPIM 19th, Ch300 p. 1632)
A. D-Dimer assay
B. 2D Echo with Doppler studies
C. Venous ultrasound of the calf
D. Chest CT with IV contrast
Diagnostic approach in VTE
Primary imaging tests for VTE
• Venous ultrasonography
• Primary criterion: loss of vein compressibility (“wink”)
• Direct visualization of thrombus
• Loss of respiratory variation in Doppler study
• Chest CT with IV contrast
• Principal imaging test for PE
• RV enlargement indicates increased risk of death within 30 days
• Can also be used to diagnose proximal leg DVT
55. Which of the following is the best known
indirect sign of PE on transthoracic
echocardiography? (HPIM 19th, Ch300 p. 1634)

A. Palla’s sign
B. Westermark’s sign
C. McConnell’s sign
D. Hampton’s hump
55. Which of the following is the best known
indirect sign of PE on transthoracic
echocardiography? (HPIM 19th, Ch300 p. 1634)

A. Palla’s sign
B. Westermark’s sign
C. McConnell’s sign
D. Hampton’s hump
Echocardiography in pulmonary embolism
• Not a reliable diagnostic imaging tool for acute PE because most
patients with PE have normal echocardiograms
• McConnell’s sign – hypokinesis of the RV free wall with normal or
hyperkinetic motion of the RV apex on transthoracic
echocardiography
• Consider doing 2D echo when CT scanning facilities are not available
or when patient has renal failure or severe contrast allergy
• Can identify saddle, R main, or L main PE
CXR in pulmonary embolism
• Normal chest x-ray often occurs
• Known abnormalities:
• Westermark’s sign – focal oligemia
• Hampton’s hump – peripheral wedge-shaped density
• Palla’s sign – enlarged right descending pulmonary artery
56. You are suspecting PE in a 68/M, bed-ridden with
prostate CA and tenderness in the calf. He has
previous anaphylactic reaction to IV contrast. What
is the appropriate diagnostic test to establish
diagnosis? (HPIM 19th, Ch300 p. 1633)
A. D-Dimer assay
B. MR pulmonary angiography
C. Ventilation-perfusion lung scanning
D. Invasive pulmonary angiography
56. You are suspecting PE in a 68/M, bed-ridden with
prostate CA and tenderness in the calf. He has
previous anaphylactic reaction to IV contrast. What
is the appropriate diagnostic test to establish
diagnosis? (HPIM 19th, Ch300 p. 1633)
A. D-Dimer assay
B. MR pulmonary angiography
C. Ventilation-perfusion lung scanning
D. Invasive pulmonary angiography
Other diagnostics in VTE
• Lung scanning
• Second-line diagnostic test, especially for those with CI to IV contrast
• Uses albumin labeled with gamma-emiting radionuclide and xenon/krypton
gas
• High-probability scan: 2 or more segmental perfusion defects in the presence
of normal ventilation
• MRI
• MR venography may be used to diagnose DVT if ultrasound is equivocal
• MR pulmonary angiography not reliable for smaller PE
Other diagnostics in VTE
• Invasive pulmonary angiography
• Chest CT with contrast has virtually replaced invasive pulmonary angiography
• Reserved for patients with technically unsatisfactory chest CTs and for those
in whom an interventional procedure such as catheter-directed thrombolysis
is planned
• Definitive diagnosis: visualization of an intraluminal filling defect in more than
one projection
57. Which of the following is the best treatment
option for a patient with Stage 4 Colon Ca who
developed DVT? (HPIM 19th, Ch300 p. 1635)

A. Low molecular weight heparin indefinitely, or until cancer free


B. Warfarin indefinitely, or until cancer free
C. Thrombolysis within 24 hours of diagnosis
D. Thrombectomy to reduce rates of postthrombotic syndrome
57. Which of the following is the best treatment
option for a patient with Stage 4 Colon Ca who
developed DVT? (HPIM 19th, Ch300 p. 1635)

A. Low molecular weight heparin indefinitely, or until cancer free


B. Warfarin indefinitely, or until cancer free
C. Thrombolysis within 24 hours of diagnosis
D. Thrombectomy to reduce rates of postthrombotic syndrome
Treatment of DVT
• Primary therapy
• Clot dissolution (low-dose catheter directed thrombolysis) – reserved for
patients with extensive femoral, iliofemoral or upper extremity DVT
• Secondary prevention
• Anticoagulation
• Inferior vena caval filter: indicated for patients who cannot be anticoagulated
or those with recurrent thrombosis despite anticoagulation
• Graduated compression stockings 30-40mmgHg for 2 years to lessen severity
of post-thrombotic syndrome
Duration of anti-coagulation
• 3 months: DVT isolated to an upper extremity or calf that has been
provoked by surgery, trauma, estrogen, or an indwelling central
venous catheter or pacemaker
• 3 to 6 months: initial episode of provoked proximal leg DVT or PE
• Indefinite: cancer (LMWH monotherapy); unprovoked DVT (including
long-haul air travel)
58. What is the major advantage of using
unfractionated heparin in VTE? (HPIM 19 ,th

Ch300 p. 1635)
A. Predictable PTT response
B. PT can be measured instead of PTT
C. Greater bioavailability
D. Short half-life
58. What is the major advantage of using
unfractionated heparin in VTE? (HPIM 19 ,th

Ch300 p. 1635)
A. Predictable PTT response
B. PT can be measured instead of PTT
C. Greater bioavailability
D. Short half-life
Heparin in pulmonary embolism
• Advantage: short half-life
• Given as initial bolus of 80u/kg then 18u/kg/hr infusion
• Target aPTT of 60-80s
• Major disadvantage is that achieving a target aPTT is empiric and may
require repeated blood sampling and heparin dose adjustment every
4-6 hours
• Risk of heparin-induced thrombocytopenia (HIT)
Low molecular weight heparin
• Greater bioavailability
• More predictable dose response
• Longer half-life than UFH
• No monitoring or dose adjustment is needed unless patient is
markedly obese or has CKD
59. 62/M rushed to the ER due to difficulty of
breathing presented with hypotension and cold,
clammy extremities. Chest CT confirmed the
diagnosis of PE. What is the appropriate treatment?
(HPIM 19th, Ch300 p. 1634 / VTE guidelines)
A. Anticoagulation alone
B. Anticoagulation with thrombolysis
C. Thrombolysis alone
D. Thrombolysis and insertion of IVC filter
59. 62/M rushed to the ER due to difficulty of
breathing presented with hypotension and cold,
clammy extremities. Chest CT confirmed the
diagnosis of PE. What is the appropriate treatment?
(HPIM 19th, Ch300 p. 1634 / VTE guidelines)
A. Anticoagulation alone
B. Anticoagulation with thrombolysis
C. Thrombolysis alone
D. Thrombolysis and insertion of IVC filter
Treatment of PE
Management of Massive PE
• Hypotension:
• Replete volume with 500ml of saline; careful hydration
• First line inotropes: dopamine and dobutamine
• Fibrinolysis:
• preferred regimen: 100 mg of recombinant tissue plasminogen activator (tPA)
administered as a continuous peripheral intravenous infusion over 2 h
• Contraindications: intracranial disease, recent surgery, and trauma
60. 45/F was admitted because of profuse vaginal
bleeding. She had a history of occasional episodic
difficulty of breathing for 1 month. PE showed
bilateral leg swelling and lower extremity
varicosities. Pertinent laboratory examination
showed Hgb = 80 mg/L, normal bleeding
parameters, unremarkable chest x-ray, elevated D-
dimer. Pelvic ultrasound showed myoma uteri.
60. Gynecology plans to do surgery for the myoma,
but only after the medical problem was managed
appropriately. Your management of the patient prior
to surgery is (HPIM 19th, Ch300 p. 1634)
A. Low molecular weight heparin
B. Fibrinolysis with rTPA (tissue plasminogen activator)
C. IVC filter insertion
D. Fresh frozen plasma transfusion
60. Gynecology plans to do surgery for the myoma,
but only after the medical problem was managed
appropriately. Your management of the patient prior
to surgery is (HPIM 19th, Ch300 p. 1634)
A. Low molecular weight heparin
B. Fibrinolysis with rTPA (tissue plasminogen activator)
C. IVC filter insertion
D. Fresh frozen plasma transfusion
Diagnosis: DVT with PE
• Principal indications for IVC filter insertion:
• Active bleeding that precludes anticoagulation
• Recurrent venous thrombosis despite intensive anticoagulation
• Soft indications for IVC filter insertion:
• Prevention of recurrent PE in patients with right heart failure who are not
candidates for fibrinolysis
• Prophylaxis of extremely high-risk patients
ARDS
61. Which of the following causes ARDS
through indirect lung injury? (HPIM 19 ,
th

Ch322 p. 1736)
A. Aspiration of gastric contents
B. Near-drowning
C. Multiple transfusions
D. Toxic inhalation injury
61. Which of the following causes ARDS
through indirect lung injury? (HPIM 19 ,
th

Ch322 p. 1736)
A. Aspiration of gastric contents
B. Near-drowning
C. Multiple transfusions
D. Toxic inhalation injury
Major causes of ARDS
• Most cases (>80%) caused by small number of clinical disorders:
• Severe sepsis syndrome and/or bacterial pneumonia (~40–50%)
• Trauma
• Multiple transfusions
• Aspiration of gastric contents
• Drug overdose
62. A previously healthy 22/M suddenly developed
difficulty of breathing several hours after getting
elbowed on the chest. CXR showed bilateral alveolar
infiltrates. PaO2 was 140 with an FiO2 of 100%. What
is the likely diagnosis? (HPIM 19th, Ch322 p. 1736)
A. Pulmonary embolism
B. Moderate ARDS
C. Pneumothorax
D. Commotio cordis
62. A previously healthy 22/M suddenly developed
difficulty of breathing several hours after getting
elbowed on the chest. CXR showed bilateral alveolar
infiltrates. PaO2 was 140 with an FiO2 of 100%. What
is the likely diagnosis? (HPIM 19th, Ch322 p. 1736)
A. Pulmonary embolism
B. Moderate ARDS
C. Pneumothorax
D. Commotio cordis
Diagnostic criteria
• Presents with severe dyspnea of rapid onset, hypoxemia, and diffuse
pulmonary infiltrates
63. Which is true regarding the
pathophysiology of ARDS? (HPIM 19 ,
th

Ch322 p. 1736)
A. Increased lung compliance
B. Hypocapnia secondary to increase in pulmonary dead space
C. Pulmonary vascular injury and vascular obliteration by
microthrombi
D. Edema fluid is not rich in protein
63. Which is true regarding the
pathophysiology of ARDS? (HPIM 19 ,
th

Ch322 p. 1736)
A. Increased lung compliance
B. Hypocapnia secondary to increase in pulmonary dead space
C. Pulmonary vascular injury and vascular obliteration by
microthrombi
D. Edema fluid is not rich in protein
Pathophysiology of ARDS
• Plasma proteins with cellular debris and dysfunctional
pulmonary surfactant aggregate in air spaces to form hyaline
membrane whorls
• Intrapulmonary shunting and hypoxemia develop
• Microvascular occlusion leads to reduction in pulmonary
arterial blood flow to ventilated portions of the lungs
(increasing dead space) and to pulmonary hypertension
• Hypercapnia secondary to an increase in pulmonary dead space
is prominent in early ARDS
64. This refers to the phase in ARDS where most
patients are liberated from ventilation and when
first signs of resolution are evident histologically?
(HPIM 19th, Ch322 p. 1737)
A. Exudative phase
B. Proliferative phase
C. Recovery phase
D. Fibrosis phase
64. This refers to the phase in ARDS where most
patients are liberated from ventilation and when
first signs of resolution are evident histologically?
(HPIM 19th, Ch322 p. 1737)
A. Exudative phase
B. Proliferative phase
C. Recovery phase
D. Fibrosis phase
ARDS – Exudative phase
• Encompasses the first 7 days of illness after exposure to a precipitating
ARDS risk factor
• Epithelial cells (Type I pneumocytes) injured  loss of tight barriers 
Cytokines + neutrophils enter interstitium and alveoli; cellular debris fill
the alveoli
• CXR: alveolar and interstitial opacities > 3/4 of lung fields
ARDS – Proliferative phase
• Day 7 to 21
• Most patients show recovery and are liberated from ventilation
• Histological signs of resolution
• Initiation of lung repair
• Shift from neutrophil to lymphocyte predominance
• Type II pneumocytes proliferate and synthesize surfactant then
differentiate into Type I pneumocytes
ARDS – Fibrotic phase
• Many patients recover lung function after 3-4 weeks but some enter
fibrotic phase
• May require long-term support on mechanical ventilators and/or
supplemental oxygen
• Extensive alveolar-duct and interstitial fibrosis; emphysema-like changes
• Consequences of structural changes:
• increased risk of pneumothorax
• reductions in lung compliance
• increased pulmonary dead space
65. You diagnose severe ARDS in a patient being
managed for CAP-HR. Which of the following
would be the best management option for your
patient? (HPIM 19th, Ch322 p. 1738)
A. Set tidal volume to 8ml/kg of predicted body weight
B. Increase PEEP to 15 mmHg
C. Perform early neuromuscular blockade with cisatracurium
D. Minimize fluids to lower left atrial filling pressures
65. You diagnose severe ARDS in a patient being
managed for CAP-HR. Which of the following
would be the best management option for your
patient? (HPIM 19th, Ch322 p. 1738)
A. Set tidal volume to 8ml/kg of predicted body weight
B. Increase PEEP to 15 mmHg
C. Perform early neuromuscular blockade with cisatracurium
D. Minimize fluids to lower left atrial filling pressures
ARDS – Management
• ARDS Network RCT: 6ml/kg of predicted body weight
• Optimal PEEP: 12-15mmHg
• Inverse-ratio ventilation (I:E > 1:1): may improve oxygenation but no
mortality benefit
• Early neuromuscular blockade: increased the rate of survival and
ventilator-free days for severe ARDS
66. Which of the following has been to
shown to demonstrate some benefit in ARDS
patients? (HPIM 19th, Ch322 p. 1738)
A. Intravenous glucocorticoids
B. Surfactant therapy
C. Inhaled epoprostenol
D. Careful diuresis
66. Which of the following has been to
shown to demonstrate some benefit in ARDS
patients? (HPIM 19th, Ch322 p. 1738)
A. Intravenous glucocorticoids
B. Surfactant therapy
C. Inhaled epoprostenol
D. Careful diuresis
ARDS – Management
Occupational lung disease
67. What is the most common cancer
associated with asbestos exposure? (HPIM
19 , Ch311 p. 1689)
th

A. Mesothelioma
B. Lung cancer
C. Nasopharyngeal carcinoma
D. Thymoma
67. What is the most common cancer
associated with asbestos exposure? (HPIM
19 , Ch311 p. 1689)
th

A. Mesothelioma
B. Lung cancer
C. Nasopharyngeal carcinoma
D. Thymoma
Asbestosis
• Diffuse interstitial fibrosing disease of the lung
• Directly related to the intensity and duration of exposure
• Encountered in individuals with only bystander exposure, such
as painters and electricians who worked alongside insulation
workers in a shipyard
• Community exposure resulted from the use of asbestos-
containing mine and mill tailings as landfill, road surface, and
playground material
Asbestos and lung cancer
• Lung CA: most common cancer associated with asbestos
• Minimum latency of 15-19 years
• Interactive effect of smoking and asbestos exposure that results
in greater risk for lung CA
Asbestos and mesothelioma
• Both pleural and peritoneal mesotheliomas are associated with
asbestos exposure
• Not associated with smoking
• Relatively short-term asbestos exposures ≤ 1-2 years or less,
occurring up to 40 years in the past
• Absolute risk of mesothelioma much less than that of lung CA
68. Which of the following findings is considered
the radiographic hallmark of asbestosis? (HPIM
19th C-311 P-1689)

A. Indistinct cardiac border


B. Uniform opacification of the pleura representing calcification
C. Irregular or linear opacities in the lower lung fields
D. Ground-glass appearance in the lung fields
68. Which of the following findings is considered
the radiographic hallmark of asbestosis? (HPIM
19th C-311 P-1689)

A. Indistinct cardiac border


B. Uniform opacification of the pleura representing calcification
C. Irregular or linear opacities in the lower lung fields
D. Ground-glass appearance in the lung fields
Radiographic findings in asbestosis

• Hallmark: irregular or linear opacities first seen in the


lower lung fields
• Other findings
• Indistinct heart border
• Ground-glass appearance in the lung fields
• Pleural plaques: thickening or calcification along the parietal
pleura  signify past exposure
69. “Crazy paving” pattern in HRCT scan is
seen in which of the following conditions?
(HPIM 19 , Ch311 p. 1690)
th

A. Asbestosis
B. Silicosis
C. Drug-induced fibrosis
D. Coal worker’s pneumoconiosis
69. “Crazy paving” pattern in HRCT scan is
seen in which of the following conditions?
(HPIM 19 , Ch311 p. 1690)
th

A. Asbestosis
B. Silicosis
C. Drug-induced fibrosis
D. Coal worker’s pneumoconiosis
Silicosis
• Chest radiograph:
• may show profuse miliary infiltration or consolidation
• Calcification of hilar nodes may occur in as many as 20% of cases (“eggshell”
pattern)
• Simple silicosis: small rounded opacities in the upper lobes may appear on the
chest radiograph
• HRCT pattern:
• multiple small nodules consistent with silicosis but also diffuse ground-glass
densities with thickened intralobular and interlobular septa producing polygonal
shapes. (“crazy paving”)
Silicosis
• The major occupational exposures include:
• Mining
• Stone cutting
• Abrasive industries such as stone, clay, glass, and cement
manufacturing
• Foundry work
• Packing of silica flour
• Quarrying, particularly of granite
• Sandblasting
Silicosis
• Silica is cytotoxic to alveolar macrophages
• Greater risk of acquiring lung infections that involve
macrophages as a primary defense (Mycobacterium
tuberculosis, atypical mycobacteria and fungi)
• Other potential complications:
• Autoimmune connective tissue disorders
• Rheumatoid arthritis
• Scleroderma
• Silica is also listed as a probable lung carcinogen
70. Which radiographic finding points to
complicated coal worker’s pneumoconiosis?
(HPIM 19 , Ch311 p. 1690)
th

A. small rounded opacities in the upper lobes


B. irregular or linear opacities in the lower lung fields
C. nodules ≥1 cm in diameter generally confined to the upper half of
the lungs
D. calcification of hilar nodes
70. Which radiographic finding points to
complicated coal worker’s pneumoconiosis?
(HPIM 19 , Ch311 p. 1690)
th

A. small rounded opacities in the upper lobes


B. irregular or linear opacities in the lower lung fields
C. nodules ≥1 cm in diameter generally confined to the upper half of
the lungs
D. calcification of hilar nodes
Coal worker’s pneumoconiosis
• Can cause chronic bronchitis and COPD
• Radiographic findings:
• Simple CWP: small, rounded opacities similar to those of silicosis  not
associated with pulmonary impairment
• Complicated CWP: nodules ≥1 cm in diameter generally confined to the upper
half of the lungs  can progress to progressive massive fibrosis and cause severe
lung function deficit
• Caplan syndrome:
• First described in coal miners, subsequently in silicosis patients
• Pneumoconiotic nodules + seropositive rheumatoid arthritis
Obstructive sleep apnea
71. OSAHS is defined as the coexistence of
unexplained excessive daytime sleepiness with at
least how many obstructed breathing events (apnea
or hypopnea) per hour of sleep?(HPIM 19th, Ch319
p. 1723)
A. 2
B. 5
C. 10
D. 15
71. OSAHS is defined as the coexistence of
unexplained excessive daytime sleepiness with at
least how many obstructed breathing events (apnea
or hypopnea) per hour of sleep?(HPIM 19th, Ch319
p. 1723)
A. 2
B. 5
C. 10
D. 15
Obstructive sleep apnea hypoventilation syndrome

• Coexistence of unexplained excessive daytime sleepiness with


>5 obstructed breathing events (apnea or hypopnea) per hour
of sleep OR AHI score > 15 with or without symptoms
• This event threshold may have to be increased in the elderly
• Apnea – breathing pauses lasting ≥ 10 seconds
• Hypopnea – events ≥ 10 seconds in which there is continued
breathing but ventilation is reduced by at least 50% from the
previous baseline during sleep
72. This refers to a ≥30% reduction in airflow for at
least 10 sec during sleep that is accompanied by
either a ≥3% desaturation or an arousal (HPIM 19th,
Ch319 p. 1725)
A. Apnea
B. Hypopnea
C. Respiratory effort-related arousal
D. Flow-limited breath
72. This refers to a ≥30% reduction in airflow for at
least 10 sec during sleep that is accompanied by
either a ≥3% desaturation or an arousal (HPIM 19th,
Ch319 p. 1725)
A. Apnea
B. Hypopnea
C. Respiratory effort-related arousal
D. Flow-limited breath
OSAHS Definitions
73. Which of the following is consistent with
the pathophysiology of obstructive sleep
apnea? (HPIM 19 , Ch319 p. 1723)
th

A. During inspiration, intraluminal pharyngeal pressure becomes


increasingly positive, creating a suctioning force
B. The airway may collapse at various levels: the soft palate (most
common), tongue base, lateral pharyngeal walls, and/or epiglottis
C. OSA may be most severe during Stage 2 sleep when neuromuscular
output to the skeletal muscles is particularly low
D. Increasing levels of O2 during sleep result in central nervous system
arousal, causing the individual to awaken
73. Which of the following is consistent with
the pathophysiology of obstructive sleep
apnea? (HPIM 19 , Ch319 p. 1723)
th

A. During inspiration, intraluminal pharyngeal pressure becomes


increasingly positive, creating a suctioning force
B. The airway may collapse at various levels: the soft palate (most
common), tongue base, lateral pharyngeal walls, and/or epiglottis
C. OSA may be most severe during Stage 2 sleep when neuromuscular
output to the skeletal muscles is particularly low
D. Increasing levels of O2 during sleep result in central nervous system
arousal, causing the individual to awaken
Pathophysiology of obstructive sleep apnea
• During inspiration, intraluminal pharyngeal pressure becomes
increasingly negative creating a suctioning force
• Reduction in neuromuscular output results in transient episodes of
airway collapse (most commonly – soft palate)
• OSA may be most severe during REM sleep when neuromuscular output
to the skeletal muscles is particularly low
• Increasing levels of CO2 during sleep result in central nervous system
arousal, causing the individual to awaken
74. Which of the following is not consistent
with the symptomatology of OSAHS? (HPIM
19 , Ch319 p. 1724)
th

A. Absence of snoring excludes the diagnosis


B. Dyspnea is an unusual symptom
C. Most common daytime symptom is true sleepiness
D. Other symptoms include a dry mouth, nocturnal heartburn,
diaphoresis of the chest and neck, nocturia, morning headaches,
trouble concentrating, irritability, and mood disturbances.
74. Which of the following is not consistent
with the symptomatology of OSAHS? (HPIM
19 , Ch319 p. 1724)
th

A. Absence of snoring excludes the diagnosis


B. Dyspnea is an unusual symptom
C. Most common daytime symptom is true sleepiness
D. Other symptoms include a dry mouth, nocturnal heartburn,
diaphoresis of the chest and neck, nocturia, morning headaches,
trouble concentrating, irritability, and mood disturbances.
Symptoms of OSAHS

• Most common symptoms:


• Snoring – absence does not exclude diagnosis
• Daytime sleepiness – does not improve with rest
• Dyspnea is an unusual symptom  differentiates OSA from PND,
nocturnal asthma and acid reflux with laryngospasm
• Predictive ability of questionnaires increase when patient is male or has
other risk factors (e.g. obesity, hypertension)
75. Which of the following is a known
clinical effect of OSAHS? (HPIM 19 ,th

Ch319 p. 1725)
A. Hypertension
B. Hypoglycemia
C. Hepatitis
D. Dyspnea at rest
75. Which of the following is a known
clinical effect of OSAHS? (HPIM 19 ,th

Ch319 p. 1725)
A. Hypertension
B. Hypoglycemia
C. Hepatitis
D. Dyspnea at rest
OSAHS - Clinical effects

• OSAHS raises 24-hour mean blood pressure


• Increase is greater in those with recurrent nocturnal hypoxemia
• At least 4-5 mm Hg
• Can be as great as 10 mm Hg in those with >20% arterial oxygen
desaturations per hour of sleep
• Increases risk of MI by around 20% and that of stroke by about 40%
OSAHS - Clinical effects

• Increased apneas and hypopneas during sleep are associated


with insulin resistance independent of obesity
• The association of OSAHS with DM is not due only to the fact
that obesity is common in both conditions
• OSAHS can aggravate diabetes; treating OSAHS in DM patients
can decrease insulin requirements
OSAHS - Clinical effects

• Hepatic dysfunction also has been associated with irregular


breathing during sleep
• Increased liver enzymes and more steatosis and fibrosis,
independent of body weight
76. Which of the following is a known risk
factor of OSAHS? (HPIM 19 , Ch319 p.
th

1725)
A. Hyperthyroidism
B. Female Sex
C. Tonsillar hypertrophy
D. Acromegaly
76. Which of the following is a known risk
factor of OSAHS? (HPIM 19 , Ch319 p.
th

1725)
A. Hyperthyroidism
B. Female Sex
C. Tonsillar hypertrophy
D. Acromegaly
OSAHS – risk factors

• The major risk factors for OSAHS are obesity and male sex.
• Additional risk factors include:
• mandibular retrognathia and micrognathia
• a positive family history of OSAHS
• Genetic syndromes that reduce upper airway patency (e.g., Down
syndrome, Treacher-Collins syndrome)
• adenotonsillar hypertrophy (especially in children)
• menopause (in women)
• various endocrine syndromes (e.g., acromegaly, hypothyroidism)
77. 52/M presents with easy fatigability and feeling
tired during the day. Sleep study confirmed
diagnosis of OSAHS. What is the current best
option for the patient? (HPIM 19th, Ch319 p. 1727)
A. Lifestyle changes including weight loss
B. Medical therapy with CPAP
C. Use of oral appliances
D. Upper airway surgery
77. 52/M presents with easy fatigability and feeling
tired during the day. Sleep study confirmed
diagnosis of OSAHS. What is the current best
option for the patient? (HPIM 19th, Ch319 p. 1727)
A. Lifestyle changes including weight loss
B. Medical therapy with CPAP
C. Use of oral appliances
D. Upper airway surgery
OSAHS – Treatment: CPAP

• Standard medical therapy with highest level of efficacy


• Improves:
• Breathing during sleep
• Sleep quality and sleepiness
• Blood pressure
• Insulin sensitivity
• Mood and alertness
• Quality of life
OSAHS – Treatment: Lifestyle
• Reduce weight
• Optimize sleep duration (7–9 hours)
• Regulate sleep schedules
• Avoid sleeping in the supine position
• Treat nasal allergies
• Increase physical activity
• Eliminate alcohol ingestion within 3 hours of bedtime
• Minimize use of sedating medications
OSAHS – Treatment: Others
• Oral appliances:
• advances the mandible, thus opening the airway by repositioning the lower jaw
and pulling the tongue forward
• AE: temporomandibular joint pain and tooth movement
• used for treating patients with mild OSAHS or patients who do not tolerate CPAP
• Upper airway surgery:
• reserved for patients who snore, have mild OSAHS, and cannot tolerate CPAP
• Most common: Uvulopalatopharyngoplasty
• less effective in severe OSAHS and in obese patients
Bronchiectasis
78. What is the imaging modality of choice to
diagnosis bronchiectasis? (HPIM 19 , Ch312
th

p. 1695)
A. Chest MRI
B. Chest X-ray
C. Chest CT scan
D. Fiberoptic bronchoscopy
78. What is the imaging modality of choice to
diagnosis bronchiectasis? (HPIM 19 , Ch312
th

p. 1695)
A. Chest MRI
B. Chest X-ray
C. Chest CT scan
D. Fiberoptic bronchoscopy
Bronchiectasis - diagnosis
• Usually based on presentation with persistent chronic cough and sputum
production accompanied by radiographic features
• CXR
• Lacks sensitivity
• Dilated airways = “tram tracks”
Bronchiectasis - diagnosis

• Chest CT scan
• More specific
• Imaging modality of choice for confirming the diagnosis
• Airway dilation
• Parallel “tram tracks”
• Signet-ring sign – a cross-sectional area of the airway at least 1.5 times that
of adjacent vessel
Bronchiectasis - diagnosis

• Chest CT scan
• Lack of bronchial tapering
• Bronchial wall thickening in dilated airways
• Inspissated secretions (“tree-in-bud” pattern)
• Cysts emanating from bronchial wall
79. Which of the following is a cause of focal
bronchiectasis? (HPIM 19 , Ch312 p. 1694)
th

A. Cystic fibrosis
B. Connective tissue disease
C. Parenchymal tumor mass
D. Idiopathic pulmonary fibrosis
79. Which of the following is a cause of focal
bronchiectasis? (HPIM 19 , Ch312 p. 1694)
th

A. Cystic fibrosis
B. Connective tissue disease
C. Parenchymal tumor mass
D. Idiopathic pulmonary fibrosis
Bronchiectasis - etiology

• Focal bronchiectasis
• Bronchiectatic changes in a localized area of the lung and can be a
consequence of obstruction of the airway (extrinsic or intrinsic)
• Extrinsic compression by adjacent lymphadenopathy or parenchymal
tumor mass
• Intrinsic obstruction by airway tumor, aspirated foreign body, stenotic
airway, or bronchial atresia from congenital underdevelopment of
airway
Bronchiectasis - etiology

• Diffuse bronchiectasis
• Characterized by widespread bronchiectatic changes throughout the
lung and often arise from an underlying systemic or infectious disease
process
Bronchiectasis - etiology
Upper Lung Fields Midlung Fields
• Cystic fibrosis • MAC infection
• Post-radiation fibrosis • Dyskinetic/immotile cilia syndrome

Lower Lung Fields Central


• Chronic recurrent aspiration • Allergic bronchopulmonary aspergillosis
• Endstage fibrotic lung disease (ABPA)
• Recurrent immunodeficiency-associated • Tracheobronchomegaly (Mounier-Kuhn
infections syndrome)
• Williams-Campbell syndrome
80. Which of the following best describes the
pathogenesis of bronchiectasis? (HPIM 19 ,th

Ch312 p. 1695)
A. British hypothesis
B. Vicious cycle hypothesis
C. Dutch hypothesis
D. Immunoparalysis
80. Which of the following best describes the
pathogenesis of bronchiectasis? (HPIM 19 ,th

Ch312 p. 1695)
A. British hypothesis
B. Vicious cycle hypothesis
C. Dutch hypothesis
D. Immunoparalysis
Bronchiectasis – pathogenesis and pathology

• “Vicious cycle hypothesis”


• Susceptibility to infection results in microbial colonization
• Some organisms like Pseudomonas exhibit propensity for
colonizing damaged airways and evading host defense
mechanisms
Bronchiectasis – pathogenesis and pathology

• Presence of microbes incites continuous chronic inflammation


and propagation of infectious/inflammatory cycle
• Mediators released from bacteria interfere with mucociliary
clearance
• Inflammation of airways results in airway obstruction
81. What is the goal of treatment in patients
with bronchiectasis? (HPIM 19 , Ch312 p.
th

1696)
A. Improvement of lung function
B. Prevention of recurrent infection
C. Control of non-tuberculous Mycobacterium (NTM)
D. Control of active infection
81. What is the goal of treatment in patients
with bronchiectasis? (HPIM 19 , Ch312 p.
th

1696)
A. Improvement of lung function
B. Prevention of recurrent infection
C. Control of non-tuberculous Mycobacterium (NTM)
D. Control of active infection
Bronchiectasis – treatment

• Treatment directed at control of active infection and


improvement in secretion clearance and bronchial hygiene so
as to decrease the microbial load within the airways and
minimize risk of repeated infections
• Treatment with antibiotics for 7-10 days
• Bronchial hygiene: hydration, mucolytics, aerosolization of
bronchodilators and hyperosmolar agents, chest physiotherapy
• Inhaled glucocorticoids
Bronchiectasis – treatment

• Patients with ABPA may benefit from prolonged course of


treatment with oral antifungal agent itraconazole
• In select cases, surgery can be considered with resection of a
focal area of suppuration
Lung abscess
82. Most nonspecific lung abscesses are
presumed to be due to which of the following
microorganisms? (HPIM 19 , Ch154 p. 814)
th

A. Mycobacteria
B. Staphylococcus aureus
C. Klebsiella pneumoniae
D. Anaerobes
82. Most nonspecific lung abscesses are
presumed to be due to which of the following
microorganisms? (HPIM 19 , Ch154 p. 814)
th

A. Mycobacteria
B. Staphylococcus aureus
C. Klebsiella pneumoniae
D. Anaerobes
Lung abscess - etiology

• Primary lung abscesses usually arise from aspiration, are often


caused principally by anaerobic bacteria, and occur in the
absence of an underlying pulmonary or systemic condition.
• S. aureus – in conjunction with influenza
• Enteric Gram (-) bacilli (esp. Klebsiella pneumoniae), Nocardia
asteroides, Rhodococcus equi – immunocompromised patients
• Mycobacterium tuberculosis is an important cause of abscess
formation
83. Where would you expect to find primary
lung abscesses? (HPIM 19th, Ch154 p. 814)

A. Right mid lower lobe


B. Left mid lower lobe
C. Right posterior upper lobe
D. Left posterior upper lobe
83. Where would you expect to find primary
lung abscesses? (HPIM 19th, Ch154 p. 814)

A. Right mid lower lobe


B. Left mid lower lobe
C. Right posterior upper lobe
D. Left posterior upper lobe
Primary lung abscess location
• In primary lung abscesses, the dependent segments (posterior upper
lobes and superior lower lobes) are the most common locations,
given the predisposition of aspirated materials to be deposited in
these areas.
• Generally, the right lung is affected more commonly than the left
because the right mainstem bronchus is less angulated
84. 55/M was admitted because of low-grade fever of 3 weeks
with cough productive of greenish foul-smelling phlegm.
Watcher claims that patient had several episodes of seizure a
month ago. PE showed gingivitis, rales of the right lung base
and clubbing. What diagnostic should be performed to clinch
the diagnosis (HPIM 19th, Ch154 p. 815)?

A. Chest x-ray PA, lateral


B. Chest CT scan
C. Chest ultrasound
D. Sputum GS/CS
84. 55/M was admitted because of low-grade fever of 3 weeks
with cough productive of greenish foul-smelling phlegm.
Watcher claims that patient had several episodes of seizure a
month ago. PE showed gingivitis, rales of the right lung base
and clubbing. What diagnostic should be performed to clinch
the diagnosis (HPIM 19th, Ch154 p. 815)?

A. Chest x-ray PA, lateral


B. Chest CT scan
C. Chest ultrasound
D. Sputum GS/CS
Lung abscess – risk factors

• Major risk factor – aspiration


• altered mental status, alcoholism, drug overdose, seizures, bulbar
dysfunction, prior cerebrovascular or cardiovascular events, or
neuromuscular disease
• colonization of the gingival crevices by anaerobic bacteria or
microaerophilic streptococci (especially in patients with
gingivitis and periodontal disease)
Lung abscess – diagnosis

• Chest CT permits better definition than CXR and earlier


evidence of cavitation
• Can provide additional information regarding a underlying cause of lung abscess,
such as malignancy, and may distinguish a peripheral lung abscess from a pleural
infection
• Sputum and blood cultures advised when a secondary lung abscess is
present or empirical therapy fails to elicit a response
85. What is the usual antibiotic of choice for
lung abscess caused by anaerobes? (HPIM
19 , Ch154 p. 815)
th

A. Levofloxacin
B. Clindamycin
C. Ertapenem
D. Metronidazole
85. What is the usual antibiotic of choice for
lung abscess caused by anaerobes? (HPIM
19 , Ch154 p. 815)
th

A. Levofloxacin
B. Clindamycin
C. Ertapenem
D. Metronidazole
Lung abscess - treatment

• For primary lung abscesses, the recommended regimens are


• (1) clindamycin (600 mg IV three times daily; then, with the disappearance of
fever and clinical improvement, 300 mg PO four times daily)
• (2) an IV-administered β-lactam/β-lactamase combination, followed—once the
patient’s condition is stable—by orally administered amoxicillin-clavulanate.
• Carbapenems – published data in the treatment of anaerobic
pulmonary infections are sparse
• Penicillin – not active against β-lactamase-producing agents
• Metronidazole – not active against microaerophilic streptococci
Lung abscess - treatment

• Duration of therapy is arbitrary


• Continuation of oral treatment until imaging shows cleared
chest lesions or small stable scar
• Persistence of fever beyond 5-7 days or progression of infiltrate
suggests failure of therapy
Lung abscess - treatment

• Failure of treatment
• Obstruction
• Complicating empyema
• Involvement of antibiotic-resistant bacteria
• Chest CT and/or bronchoscopy to exclude other pathologies
86. 55/M was admitted because of low-grade fever of 3 weeks
with cough productive of greenish foul-smelling phlegm. PE
showed rales of the right lung base and clubbing. Chest x-ray
showed cavity with an air fluid level at the right base. The
patient was given 6 weeks of clindamycin but symptoms
persisted and x-ray findings remained the same. Management
of this patient is : (HPIM 19th, Ch154 p. 815)
A. Shift antibiotics and treat for another 4-6 weeks
B. Treat as pulmonary tuberculosis
C. Surgery on lesion
D. Anti-fungal treatment
86. 55/M was admitted because of low-grade fever of 3 weeks
with cough productive of greenish foul-smelling phlegm. PE
showed rales of the right lung base and clubbing. Chest x-ray
showed cavity with an air fluid level at the right base. The
patient was given 6 weeks of clindamycin but symptoms
persisted and x-ray findings remained the same. Management
of this patient is : (HPIM 19th, Ch154 p. 815)
A. Shift antibiotics and treat for another 4-6 weeks
B. Treat as pulmonary tuberculosis
C. Surgery on lesion
D. Anti-fungal treatment
Lung abscess – indications for surgery
• Reserved for 10-12% of patients
• Major indications:
• Failure to respond to antibiotics
• Obstructed bronchus
• Large abscess (>6 cm)
• Resistant bacteria
• Suspected neoplasm
• Hemorrhage
• Usual procedure is lobectomy
• Alternative intervention – percutaneous drainage under CT guidance
Interstitial lung disease
87. What is the main cause of hypoxemia in
patients with ILD? (HPIM 19 , Ch315 p.
th

1709)
A. Shunting
B. Mismatching of ventilation to perfusion
C. Increased DLCO
D. Decreased blood flow to well-ventilated areas
87. What is the main cause of hypoxemia in
patients with ILD? (HPIM 19 , Ch315 p.
th

1709)
A. Shunting
B. Mismatching of ventilation to perfusion
C. Increased DLCO
D. Decreased blood flow to well-ventilated areas
Hypoxemia in ILD
• Decreased DLCO common, partly due to effacement of alveolar
capillary units
• Mismatching of ventilation and perfusion
• Reduced compliance results in poor ventilation but maintained
blood flow
88. Which of the following best describes the
pathogenesis of interstitial lung disease?
(HPIM 19 , Ch315 p. 1708)
th

A. Mostly caused by identified infectious agents


B. The major histologic pattern is calcified granuloma formation
C. The initial insult is injury to the epithelial surface which causes
inflammation and eventually fibrosis
D. Chronic scarring of the airways is usually reversible
88. Which of the following best describes the
pathogenesis of interstitial lung disease?
(HPIM 19 , Ch315 p. 1708)
th

A. Mostly caused by identified infectious agents


B. The major histologic pattern is calcified granuloma formation
C. The initial insult is injury to the epithelial surface which causes
inflammation and eventually fibrosis
D. Chronic scarring of the airways is usually reversible
Pathogenesis of interstitial lung disease
• ILDs are nonmalignant disorders and are not caused by
identified infectious agents
• Two major histopathologic patterns:
• granulomatous pattern
• Inflammatory pattern with fibrosis
ILD Symptoms
• Progressive exertional dyspnea
• Persistent nonproductive cough
• Hemoptysis
• Wheezing
• Chest pain
Lung Response: Alveolitis, Interstitial
Inflammation, and Fibrosis
• Known Cause • Unknown Cause
• Asbestos • Idiopathic interstitial
• Fumes, gases pneumonias (IPF, etc.)
• Drugs (antibiotics, amiodarone, • CTDs (SLE, RA, AS, etc.)
gold, chemo drugs) • Pulmonary hemorrhage
• Radiation syndromes
• Aspiration pneumonia • Pulmonary alveolar proteinosis
• Residual of ARDS • Eosinophilic pneumonias
• Smoking-related • Amyloidosis
• Inherited diseases (tuberous
sclerosis, NF, etc.)
• GI or liver diseases
• Graft vs. host disease
Lung Response: Granulomatous
• Known Cause
• Hypersensitivity pneumonitis (organic dusts)
• Inorganic dusts (beryllium, silica)
• Unknown Cause
• Sarcoidosis
• Granulomatous vasculitides
• Wegener’s granulomatosis, Churg-Strauss Syndrome
• Bronchocentric granulomatosis
• Lymphomatoid granulomatosis
ILD Groups
• Most common ILDs of unknown etiology
• Sarcoidosis
• Idiopathic Pulmonary Fibrosis (IPF)
• Pulmonary fibrosis associated with CTDs
• ILDs of known causes
• Occupational and environmental exposures
• Inhalation of inorganic dusts and various fumes or gases
Community-acquired
Pneumonia
Clinical Practice Guidelines
89. 48/M known diabetic with good glycemic control,
presents to the ER for cough and fever. On PE: HR 136 T
39.2 RR 32 BP 120/70, coarse crackles, bilateral. CBG:
280. CXR showed bilateral infiltrates. What is the
appropriate course of management (CPG – CAP 2016):
A. Start patient on oral antibiotics, send home from ER
B. Transfer patient to ambulatory care, start oral antibiotics and
supplemental insulin before sending home
C. Admit patient to medical ward and give intravenous antibiotics
D. Admit patient to ICU and give intravenous antibiotics
89. 48/M known diabetic with good glycemic control,
presents to the ER for cough and fever. On PE: HR 136 T
39.2 RR 32 BP 120/70, coarse crackles, bilateral. CBG:
280. CXR showed bilateral infiltrates. What is the
appropriate course of management (CPG – CAP 2016):
A. Start patient on oral antibiotics, send home from ER
B. Transfer patient to ambulatory care, start oral antibiotics and
supplemental insulin before sending home
C. Admit patient to medical ward and give intravenous antibiotics
D. Admit patient to ICU and give intravenous antibiotics
Risk stratification
LOW MODERATE HIGH
Stable vital signs: Unstable vital signs: Moderate risk + any of the
RR < 30 RR > 30 following:
HR < 125 HR > 125 Severe sepsis/ Septic shock
SBP > 90 , DBP > 60 SBP < 90 , DBP < 60 OR Need for mechanical
Temp > 36 or < 40 Temp < 36 or > 40 ventilation
No altered mental state of acute Altered mental state of acute onset
onset
No suspected aspiration Suspected aspiration
No or stable co-morbid condition Unstable co-morbid condition (uncontrolled DM;
active malignancy; neurologic disease in
evolution; CHF FC II-IV; unstable CAD; renal
failure on dialysis; uncompensated COPD;
decompensated liver disease)
CXR: localized infiltrates; no CXR: bilateral infiltrates, multilobar disease,
evidence of pleural effusion evidence of pleural effusion
OUTPATIENT INPATIENT INPATIENT - ICU
90. 48/M known diabetic with good glycemic control,
consults for fever (Tmax 38.3), cough and weakness. Rest
of vitals are within normal limits. X-ray shows localized
infiltrates on the right lower lobe. What is the appropriate
treatment regimen (CPG – CAP 2016):
A. Amoxicillin 1g TID x 5-7 days
B. Azithromycin 500mg OD x 3-5 days
C. Co-amoxiclav 1g BID x 5-7 days
D. Cefixime 400mg BID + Clarithromycin 500mg BID x 5-7 days
90. 48/M known diabetic with good glycemic control,
consults for fever (Tmax 38.3), cough and weakness. Rest
of vitals are within normal limits. X-ray shows localized
infiltrates on the right lower lobe. What is the appropriate
treatment regimen (CPG – CAP 2016):
A. Amoxicillin 1g TID x 5-7 days
B. Azithromycin 500mg OD x 3-5 days
C. Co-amoxiclav 1g BID x 5-7 days
D. Cefixime 400mg BID + Clarithromycin 500mg BID x 5-7 days
91. Which of the following is the least appropriate
approach in handling patients who are not recovering after
72 hours of empiric antibiotic therapy(CPG – CAP 2016):

A. Shift to alternative antibiotics immediately


B. Review all diagnostic results and reassess for presence of other
pathogens
C. Do repeat chest radiography
D. Consider obtaining additional specimens for microbiologic testing
91. Which of the following is the least appropriate
approach in handling patients who are not recovering after
72 hours of empiric antibiotic therapy(CPG – CAP 2016):

A. Shift to alternative antibiotics immediately


B. Review all diagnostic results and reassess for presence of other
pathogens
C. Do repeat chest radiography
D. Consider obtaining additional specimens for microbiologic testing
Not responding to treatment after 72h
• Complete reappraisal  NOT simply shifting antibiotics
• Consider:
• Possible resistance to antibiotics
• Other pathogens: TB, virus, parasites, fungi
• Do follow-up chest radiography to investigate for:
• Pneumothorax, cavitation, pleural effusion, extension to other lobes,
pulmonary edema, ARDS
• Do CT scan for:
• Underlying mass, bronchiectasis, loculation, abscess
• Consider repeat microbiologic testing
92. When is a follow-up chest xray recommended for
patients recovering from CAP(CPG – CAP 2016):

A. Every 3-5 days while hospitalized


B. Prior to hospital discharge
C. After 4-6 weeks post hospital discharge
D. A follow-up chest xray is not recommended for patients who
recovered from CAP
92. When is a follow-up chest xray recommended for
patients recovering from CAP(CPG – CAP 2016):

A. Every 3-5 days while hospitalized


B. Prior to hospital discharge
C. After 4-6 weeks post hospital discharge
D. A follow-up chest xray is not recommended for patients who
recovering from CAP
Questions?
PULMONARY
MEDICINE
Joselito R. Chavez, MD
Aaron Joseph K. Tiu, MD

You might also like